Chest and Abdominal Wall, Gyn

Réussis tes devoirs et examens dès maintenant avec Quizwiz!

Which of the following best represents the likelihood of malignancy in adolescents undergoing subcutaneous mastectomy for gynecomastia? A ) 1%B ) 5%C ) 10%D ) 15%

The correct response is Option A. A literature search yielded over 2000 articles in total; however, only 36 articles have discussed cases of adolescent gynecomastia and the associated pathologic results, resulting in data for 615 individuals. Of these 615 individuals, there have been six cases of cancer and five cases of atypical ductal hyperplasia associated with adolescent gynecomastia. Specific patient information was only available for seven of the individuals (six breast cancer and one atypical ductal hyperplasia), which revealed that the average age of patients involved was age 17.4 years (range, age 16 to 20 years), 43% of cases had symptoms of unilateral gynecomastia, and an abnormal physical examination was present in only one case. In the cases with histologic characterization, five cases were ductal carcinoma in situ with low to intermediate grades, and the other case had been diagnosed as invasive carcinoma of the secretory carcinoma type. Sixty percent of the cases of ductal carcinoma in situ had associated atypical ductal hyperplasia as well.

Gynecomastia that occurs in men who take anabolic steroids is caused by increased levels of which of the following? A) AromataseB) EstradiolC) High-density lipoproteinD) Testosterone

The correct response is Option B. Anabolic steroids have sex-specific adverse effects. Development of breast tissue in men, a condition called gynecomastia (which is usually caused by high levels of circulating estradiol), may arise because of increased conversion of testosterone to estradiol by the enzyme aromatase. Decreased sexual function and temporary infertility can also occur in men.

A 60-year-old man undergoes sigmoid resection and colostomy for management of ruptured diverticulitis. The patient has smoked one pack of cigarettes daily for the past 35 years. BMI is 36 kg/m2. After colostomy reversal, he has an abdominal wound infection and fascial dehiscence. Reconstruction with a bridging human acellular dermal matrix is planned. Compared with traditional polypropylene mesh repair, which of the following complications is more likely with the planned approach? A) Abdominal bulgeB) FistulaC) HematomaD) InfectionE) Skin necrosis

The correct response is Option A. Acellular dermal matrices (ADM) have been advocated for the past decade as an important adjunct in the complex field of abdominal wall reconstruction. Many studies have verified the use and general safety of ADM in abdominal wall reconstruction, but conclusive evidence of its advantages over other techniques is still lacking. What can be inferred is its advantage over prosthetic mesh in contaminated fields. Polypropylene mesh would be contraindicated in the infected wound in this example. After adequate debridement and appropriate antibiotics in an optimized patient, ADM can be used for hernia repair or reconstruction along with component separation. Postoperative infections can be as common as 40%, but conservative management measures often suffice, rather than reoperation and graft explantation, which are required with prosthetic mesh. One recognized drawback in the stretchable nature of dermal matrix grafts is that they can often stretch under tension to 50% or more of their initial dimensions. Postoperative bulging without true herniation is common. Strategies for prevention include suturing the graft under maximal stretch and use of porcine dermal grafts rather than human grafts. Hernia recurrence in the complex abdominal wall reconstruction remains a common event, regardless of technique. Although some studies assert a strong advantage with ADM, others report a similar or increased recurrence rate. As successful operative techniques become more standardized, perhaps more uniform success will be demonstrated in future studies. Skin necrosis and hematoma are common surgical complications that should not vary between choice of graft material. Fistula rates are decreased with ADM versus prosthetic mesh reconstruction.

An otherwise healthy 35-year-old man comes to the office because of a 15-year history of bilateral Grade III gynecomastia. Height is 5 ft 10 in (178 cm) and weight is 187 lb (85 kg). BMI is 27 kg/m2. Physical examination shows breast enlargement with skin redundancy and palpable glandular and fatty tissue. Ultrasound-assisted lipectomy is planned because the patient wishes to avoid visible scars. Which of the following is most likely to minimize the need for skin resection? A ) Disruption of the inframammary foldB ) High energy settingC ) Peripheral feathering with standard liposuctionD ) Treatment of the intermediate fat layerE ) Tumescent infiltration with a 3:1 ratio

The correct response is Option A. Although surgery remains the mainstay of treatment for gynecomastia in this country, results have been inconsistent, with reported dissatisfaction rates as high as 50%. Common aesthetic problems reported after surgery include unacceptable scarring and nipple and areolar deformities such as tethering and malposition. As a result, less invasive techniques have evolved and are now the mainstay of treatment for all but the most severe forms of gynecomastia. The use of ultrasound-assisted lipectomy has improved the outcomes in the treatment of all grades of gynecomastia. In the more severe grades, certain maneuvers will improve skin retraction and redraping and may thus eliminate the need for a subsequent procedure for skin excision. While fatty and fibrous tissue is best approached from a deeper subcutaneous plane, transition to a subdermal plane can allow for greater skin retraction. Moreover, disruption of the inframammary fold is essential in that it allows the skin to drape more naturally onto the abdomen, which is typical of a male breast. The use of wetting solutions is imperative with ultrasound-assisted lipectomy to avoid burns. Both superwet (1:1) and tumescent (3:1) techniques are acceptable. The power of the unit should generally not exceed 90% to avoid burns and dermal injuries. Peripheral feathering improves contour, providing a smoother transition to the outer border of the breast, but does not specifically address skin excess.

A 48-year-old man undergoes revision ventral hernia repair. Medical history includes poorly controlled type 1 diabetes mellitus and liver transplantation 3 years ago. The duration of the operation is 6 hours and the procedure is complicated by extensive lysis of adhesions. Three days postoperatively, the patient is noted to have decreased urine output and hypotension despite crystalloid and colloid fluid resuscitation. Intrinsic renal failure is suspected. On analysis of serum and urinary electrolytes, which of the following values is expected to be normal with intrinsic renal failure in this patient? A) BUN:serum creatinine ratioB) Fractional excretion of sodium (FENa)C) UrinalysisD) Urinary sodium concentrationE) Urine osmolality

The correct response is Option A. Assessment of renal failure begins with a thorough history and physical examination as well as evaluation of key laboratory measurements including complete blood count, metabolic panel, coagulation profile, urinalysis, and urine electrolytes. Early signs of renal failure may include orthostatic hypotension, tachycardia, and dry mucous membranes on examination. Renal failure may be divided into three categories: prerenal (causes include hypovolemia, cardiac failure, sepsis), renal/intrinsic (causes include disorders of the renal parenchyma, glomerular disease, acute tubular necrosis), and postrenal (causes included renal vein occlusion, urinary tract obstruction). Assessment of serum and urinary electrolytes and a urinalysis aid with the identification of the cause of renal failure. In prerenal failure, the urinalysis is normal, the ratio of blood urea nitrogen (BUN) to serum creatinine is elevated above 20:1, the urinary sodium concentration is less than 20 mEq/L, urine osmolality is higher than 500 mOsmol/kg H2O, and the fractional excretion of sodium (FENa), which compares the differences of the sodium and creatinine in the plasma and urine, is less than 1%. Intrinsic renal failure usually presents with a normal BUN and serum creatinine, urinary sodium concentration higher than 40 mEq/L, urine osmolality less than 350 mOsmol/kg H2O, and an abnormal urinalysis. Postrenal failure typically has an elevated BUN to serum creatinine, a FENa greater than 1%, and a normal urinalysis.

The blood supply to the mid-abdomen originates primarily from which of the following arcade regions? A ) Deep epigastricB ) Deep internal mammaryC ) Mid epigastricD ) Superficial epigastricE ) Superficial internal mammary

The correct response is Option A. Blood is supplied to the abdominal wall both by direct cutaneous vessels and musculocutaneous vessels. Haddad demonstrated the exact locations of these vessels using arteriography. Huger later classified the vascular supply into simple zones for use in abdominal lipectomy. This classification can be applied to abdominal reconstruction. The mid-abdomen is supplied primarily by the deep epigastric arcade. The superior epigastric artery arises from the internal thoracic artery and descends with the posterior rectus sheath. The deep inferior epigastric artery arises from the external iliac artery and ascends with the posterior rectus sheath. Each of these vessels supplies branches to the rectus muscle and overlying skin. The lower abdomen is supplied by branches of the epigastric arcade and the external iliac artery. The superficial epigastric and superficial external pudendal arteries originate from the femoral artery running superficial to the fascia to supply the overlying skin. The inferior epigastric artery on the posterior rectus sheath supplies the underlying muscle in this zone. The deep circumflex iliac artery supplies the muscles in the area of the anterior iliac spine. The flanks and lateral abdomen are supplied by the intercostal, subcostal, and lumbar arteries. These vessels arise from the aorta and give off perforators to the back and flank. They travel circumferentially on the transversus abdominis, perforating the oblique muscles and supplying the overlying skin.

A 2-year-old boy with a history of omphalocele presents for correction of a 5-cm abdominal bulge with a 3-cm widened scar over the bulge. Which of the following is the most appropriate next step in management of the bulge? A) Anterior component separationB) Interposition acellular dermal matrix placementC) Interposition prosthetic mesh placementD) Tensor fascia lata flapE) Tissue expander placement

The correct response is Option A. For most patients with omphaloceles less than 5 cm in diameter, a single operation involving a traditional anterior component separation from costal margin to iliac crest is sufficient to reduce the omphalocele and reapproximate the rectus diastasis. Extended component separations are typically only needed when the omphalocele is large and accompanied by a diaphragmatic hernia. Once the abdominal wall defect extends past 5 cm in diameter, a staged procedure involving the placement of tissue expanders and subsequent flap advancement must be considered. Autologous tissue options, such as the tensor fascia lata flap, may be needed for larger defects that can not be managed with component separation. While prosthetic or biologic mesh placement is used to correct large abdominal wall defects, it is typically used only after autologous options have failed or are not available.

Which of the following is the most common hyperplastic childhood breast anomaly? A ) Giant fibroadenomaB ) GynecomastiaC ) Juvenile hypertrophyD ) PolymastiaE ) Polythelia

The correct response is Option B. Breast anomalies in children are a relatively common finding. A recent study showed that hyperplastic abnormalities were the most common category, with the most common anomalies being gynecomastia, followed by juvenile hypertrophy or hyperplasia. The average age at initial surgery for the group was age 17.4 years; the average number of operations per patient was 1.14. Patients most likely to require revisional surgery were those with giant fibroadenoma.

A 55-year-old man comes to the office because of the ventral hernia shown. History includes a sigmoid colectomy for diverticular disease using a midline incision 7 years ago. Postoperatively, an incisional hernia developed. He underwent mesh repair, which was unsuccessful. Physical examination shows a 28 (horizontal at the level of the umbilicus) × 25-cm (vertical) palpable fascial defect. Which of the following is the most appropriate method of reconstruction of the abdominal wall?A ) Component separation technique only B ) Component separation technique with mesh repair C ) Mesh repair only D ) Tissue expansion and component separation technique with mesh repair E ) Use of a pedicled tensor fascia lata flap

The correct response is Option B. Component separation is a well-described technique for repairing midline abdominal wall hernias. Originally described by Ramirez et al. in 1990, the technique involves incising the external oblique fascia lateral to the rectus muscle. The external oblique is elevated off of the internal oblique muscle in a loose areolar plane that is largely avascular, except for the intercostal branches to the external oblique muscles that are located far posteriorly. The posterior rectus sheath can be incised longitudinally to provide an additional 1 to 2-cm mobilization if needed. The attachments of the internal oblique to the rectus muscle are preserved, as are the intercostal neurovascular bundles that supply the rectus muscle. Hence, the rectus muscle is kept innervated and functional. Under ideal conditions, mobilization of the rectus-internal oblique muscle flap on both sides allows for closure of defects at the epigastrium, mid abdomen, and low abdomen of 10, 20, and 6 cm, respectively. A component separation repair alone would be insufficient because the defect described is 28 cm wide. There would still be a persistent defect in the fascia. Therefore, component separation with supplemental mesh would be required to adequately close this defect. Component separation mobilizes functional muscle medially to help restore abdominal wall integrity and minimizes the amount of bridging that would be repaired with mesh. A mesh repair alone would close the fascial defect but would do nothing to restore the functionality of the abdominal wall. Large defects repaired with mesh can be associated with hernia recurrence and long-term bulging. Tissue expansion can be useful in situations where skin coverage may be lacking. Tissue expansion is unnecessary in the patient described because skin coverage is not an issue. Free flap reconstruction, such as with a tensor fascia lata flap, has been described for abdominal wall defects and can provide vascularized fascia. There are even case reports of innervated free flap reconstructions. However, failure to maintain sufficient muscular tone in the long term leads to eventual attenuation of the fascia and abdominal wall bulging.

An otherwise healthy 47-year-old man comes to the office because of the recurrent ventral hernia shown. He does not smoke cigarettes. Physical examination shows "Swiss cheese-type" defects, which are confirmed by CT scan. He desires reconstruction. Which of the following is the most appropriate surgical treatment? A) Component separation with mesh placementB) Laparoscopic hernia repair with mesh placementC) Open hernia reduction with bridging mesh placementD) Open reduction with free tensor fascia lata flapE) Total autologous component separation

The correct response is Option A. Given the size and history of the defect, the patient described has a significant chance of recurrence of his hernia. To optimize the chances of a functional recovery that is durable and has the lowest chance of recurrence, the component separation technique with mesh reinforcement (synthetic or biologic) is indicated. The component separation technique, originally described by Ramirez et al. in 1990, allows for recruitment of the rectus complex towards the midline to facilitate primary reapproximation (in the best-case scenario) or at least decrease the size of the residual defect. Studies have shown that reinforcement of hernia repairs with mesh decreases recurrence rates by 50 to 75%, even in secondary repairs. It is not clear, however, whether the mesh should be placed as an overlay or underlay. Nonetheless, mesh reinforcement leads to lower recurrence rates, especially if combined with component separation. Specifically, component separation allows for reduction of the hernia defect size after hernia reduction, and leads to lower recurrence rates versus hernia reduction and bridging mesh repair alone. Laparoscopic repair is a useful technique in hernia repairs. However, the complex, large "Swiss cheese-type" defect described lends itself more to open repair in terms of facilitation of reduction, optimization of safety (avoidance of unrecognized bowel injury), and decrease in operative time. A tensor fascia lata flap, originally described for large hernia repairs because it allows transfer of fascia to the abdomen, does not demonstrate superior results compared to available meshes. It also incurs donor site morbidity versus the mesh reinforcement techniques.

A 59-year-old woman presents with an infected sternal nonunion after coronary artery bypass grafting 4 weeks ago. After debridement of the wound, five sternal plates and bilateral pectoralis flaps are placed. Postoperatively, the patient becomes hypotensive, tachycardic, and confused. Jugular distention is noted. Oxygen saturation is 100% on nasal cannula. Which of the following is the most appropriate initial step in management? A) AuscultationB) Chest x-rayC) ECGD) Ultrasonography of the heartE) Return the patient to the operating room

The correct response is Option A. On auscultation a muffled heart sound and pericardial friction rub is heard and would direct the clinician to decompress tamponade. Patient is demonstrating Beck's triad and has reason for possible cardiac tamponade. Immediate chest x-ray can be ordered to help rule out pneumothorax, but with normal oxygenation, the chance of a pneumothorax is lower on the differential, and there are other better initial diagnostic and therapeutic steps. ECG can help support the diagnosis of pericardial effusion, but this is not diagnostic and is only used as an adjunct. Ultrasonography of the heart can confirm the existence of pericardial effusion, as well as allow needle drainage for immediate treatment. However, this would be performed after auscultation.

A 70-year-old man is evaluated because of chest wall incision drainage associated with leukocytosis, high fever, and blood cultures positive for Staphylococcus aureus 14 days after undergoing quintuple coronary artery bypass grafting using both internal mammary arteries. History includes type 2 diabetes mellitus, hypertension, hypercholesterolemia, and chronic obstructive pulmonary disease. He has smoked one pack of cigarettes daily for the past 40 years. BMI is 32 kg/m2. In addition to broad-spectrum antibiotic therapy, which of the following is the most appropriate initial management of this condition? A) Debridement and negative pressure wound therapyB) Debridement and reconstruction with a pectoralis turnover flapC) Debridement and reconstruction with sternal platingD) Debridement with primary rewiring and wound irrigation

The correct response is Option A. Post-sternotomy mediastinitis is a severe complication, with an incidence ranging between 0.2 and 8%. Risk factors include advanced age, diabetes mellitus, morbid obesity, reoperation for bleeding, and use of bilateral internal mammary arteries. Deep sternal wound infections are more serious and have a higher mortality rate than superficial or sterile sternal wounds. The most common organisms are Staphylococcus aureus and Staphylococcus epidermidis, but mixed gram-positive and gram-negative infections are not uncommon. Broad-spectrum antibiotic therapy to include coverage of MRSA infection and Pseudomonas should be instituted. Nevertheless, debridement is the mainstay of therapy. Adequate sternal and soft-tissue debridement is vital and can be combined with immediate sternal closure with delayed primary wound closure versus delayed closure of the entire wound with or without flaps to aid in obliterating any dead space. Many studies have demonstrated that the use of negative pressure wound therapy for wound coverage as a bridging method to final wound closure decreases the morbidity of these patients and is associated with decreased recurrent infection and treatment failure rates compared with conventional therapy, specifically primary rewiring or plating, especially for those at high risk. Flap closure at the time of sternal debridement can be performed; however, if bilateral internal mammary arteries are used for the bypass, a pectoralis turnover flap is not an option because of loss of the internal mammary artery for the pectoralis major. Options would include coverage with the pectoralis transposition flap based on the thoracoacromial artery, an omental flap, or a rectus abdominis flap based on the intercostal artery.

A 65-year-old woman presents to the office with an ulcer on the right chest wall. She underwent right-sided mastectomy and adjuvant external beam radiation therapy for advanced breast cancer 5 years ago. Physical examination shows a 2-cm ulcer with surrounding radiation-damaged skin and no signs of acute infection. Which of the following is the most appropriate next step in management? A) Biopsy of the woundB) Excision of all radiation-damaged tissue and coverage with vascularized tissueC) Excision of the ulcer and coverage with vascularized tissueD) Hyperbaric oxygen therapyE) Negative pressure therapy

The correct response is Option A. Radiation causes production of reactive oxygen species, which causes injury to tissues and progenitor cells. Cytokine release results in chronic inflammation and ongoing tissue damage. Radiation therapy can cause soft-tissue ulcerations, osteoradionecrosis, and radiation-induced sarcomas. If a patient presents with a late ulcer after radiotherapy, malignancy needs to be ruled out. A biopsy of the ulcer edge should be performed. Once malignancy has been ruled out, excision of all radiation-damaged tissue, rather than just the ulcer, will result in more durable reconstructive outcomes. Osteoradionecrosis of the chest wall presents as full-thickness chest wall ulcers and the involved ribs should be resected. The underlying pleura and lung may be adherent and, thus, limited lung resection may need to be performed. Reconstruction is performed with well-vascularized tissue, either local pedicled flaps or free flaps. Negative pressure therapy utilizes subatmospheric pressure for local wound care. It provides local wound care by controlling exudate and, thus, keeping the wound clean. It is thought to promote wound healing by inducing cellular proliferation and increasing capillary blood flow. Malignancy in the wound is a contraindication to negative pressure therapy. Therefore, if suspected, malignancy should be ruled out prior to initiation of negative pressure therapy. Hyperbaric oxygen is the administration of 100% oxygen in a pressurized chamber. This results in high tissue concentrations of oxygen, which promote neovascularization and wound healing. Hyperbaric oxygen has been shown to improve healing in soft-tissue radionecrosis and osteoradionecrosis. It can be used as an adjunct, especially when radical excision and reconstruction of radiation damaged tissue is not possible.

Which of the following represents the lateral border of the breast footprint? A) Anterior axillary lineB) Anterior edge of the latissimus dorsi muscleC) Lateral clavicleD) Midaxillary lineE) Posterior axillary line

The correct response is Option A. The lateral breast is mobile, unlike the inframammary fold. Therefore, the footprint can vary slightly and be just behind the anterior axillary fold, but it does not extend to the midaxillary line. The breast footprint is not described as relating to either the latissimus dorsi or the clavicle.

A 36-year-old man is scheduled to undergo correction of grade IV gynecomastia. On physical examination, the distance from the nipple to the sternal notch is 25 cm and the inframammary crease is 4 cm inferior to the lower pectoral border. A mastectomy and free nipple grafting are planned. Which of the following is the most appropriate configuration and position of the new nipple-areola complex? A ) Oval; placement at the fourth-to-fifth intercostal space B ) Oval; placement 20 cm from the midclavicle C ) Oval; placement 25 cm from the sternal notch D ) Round; placement 25 cm from the sternal notch E ) Round; placement at the fourth-to-fifth intercostal space

The correct response is Option A. The number of male body contouring procedures performed in the United States has been steadily increasing over recent years, and correction of high-grade gynecomastia remains one of the most challenging procedures performed by plastic surgeons today. A grading system for gynecomastia was originally described in 1973 and later modified in 2003. Pseudogynecomastia after massive weight loss, in which the redundant breast tissue consists primarily of fat and skin, has been separately classified, and a grading system was published in 2008. Although there are minor differences between the varying classification schemas, it is generally agreed that high-grade gynecomastia, with severe ptosis and a large amount of redundant skin, may require skin resection and movement of the nipple-areola complex using either a pedicled or free nipple grafting technique. The proper position, shape, and viability of the nipple-areola complex remain the major factors in successful outcome after mastectomy and free nipple grafting in the correction of gynecomastia. Many studies have evaluated these parameters with results that are fairly consistent, although recommendations can be somewhat formulaic and cumbersome. In the simplest of schema, 100 normal male volunteers were measured to determine the appropriate anatomical parameters for nipple position and areolar diameter. It was found that patient height was the most important fixed parameter from which nipple position should be calculated. The average diameter of the nipple-areola complex in men was found to be 28 mm, and the sternal notch-to-nipple distance was found to be 20 cm. These numbers may, however, apply less in the setting of obesity or severe weight loss. One consistent finding with respect to nipple-areola configuration in men is the shape. In one study of 100 healthy male subjects performed in 2001, it was found that 91% had an oval-shaped complex with a mean ratio of horizontal-to-vertical diameter of 27:20 mm. The mean nipple-areola diameter of the 9% of patients who had a round complex was 2.3 cm. It was also found that the center of the nipple-areola complex was in the fourth intercostal space in 75% of patients and in the fifth intercostal space in 23% of patients. Unsatisfactory results after surgical correction of gynecomastia are largely secondary to improper nipple placement, with the nipple-areola complex being placed too low and close to the sternum. Moreover, to avoid a €œstuck-on € appearance, use of a small oval, rather than a perfectly round shape, will result in a more natural appearance.

An 80-year-old man comes to the office because of recurrent squamous cell carcinoma of the cervical skin. Following reconstruction with a pectoralis myocutaneous flap, the distal half of the skin paddle appears ischemic and eventually exhibits necrosis and full-thickness skin loss. Transection of which of the following structures is most likely responsible for this complication? A) Lateral thoracic arteryB) Medial pectoral veinC) Posterior intercostal vasculatureD) Thoracodorsal arteryE) Transverse cervical vessel

The correct response is Option A. The pectoralis major muscle has three major blood supplies. In general, the internal mammary perforators and the thoracoacromial vessels are dominant, with additional perfusion through the lateral thoracic artery. Turnover pectoralis flaps for sternal reconstruction based on the internal mammary perforators are generally well perfused. Likewise, in most situations a pedicled myofascial or myocutaneous flap based on the thoracoacromial vessels (with the other blood supplies ligated) is robust and can be used for most types of head and neck reconstruction. Although described earlier, the flap became the "workhorse" of head and neck reconstruction after Ariyan's classic article in 1979. Versus a free tissue transfer, the pectoralis flap has many detractors, including being tethered to its pedicle. In about 6% of cases, the lateral thoracic vessel is the dominant pedicle to the flap and needs to be incorporated for maximal perfusion for head and neck reconstruction. This of course further decreases some of the mobility of the flap. This phenomenon is usually easily discernible by the larger caliber of the lateral thoracic vessels versus the thoracoacromial vessels. In this case, this was the most likely injured vessel, causing the described outcome. The transverse cervical vessels do supply the posterior thorax, and the intercostal vessels do supply the anterior chest, including breast tissue, but neither gives important perfusion to the pectoralis flap. The lateral pectoral nerve travels with the thoracoacromial vessels and, if not checked when turning the flap, can kink the vascular pedicle and, if not severed, does not allow the flap to decrease in size because of lack of denervation of the muscle. The medial pectoral nerve is another innervation to the pectoralis major muscle and is usually severed during flap elevation. The thoracodorsal artery does not supply the pectoralis muscle.

A 30-year-old woman is referred for consultation because of a 3-year history of pain and irritation of the labia minora when she exercises. She now has enlarged, protuberant labia. Labioplasty is planned. Which of the following postoperative adverse outcomes is most likely in this patient? A ) Group B streptococcal infection B ) Incomplete correction C ) Keloid formation D ) Painful intercourse

The correct response is Option B. Labioplasty is a common surgical procedure used to reduce the size of excessively protruding or enlarged labia. The condition can result in discomfort or irritation with exercise or intercourse. Frequently, surgery is sought for cosmetic reasons alone. The goal of surgery is to reduce the size of the labia so that they become closer to the level of, or posterior to, the labia majora. Direct amputation-excision should be avoided, as an anterior scar line can cause postoperative discomfort in clothing or with sexual activity. Effective surgical methods use incisions that place the scars inferior, interior, or transverse, such as the inferior wedge resection with superior flap technique. When the surgery is performed properly, long-term sexual complaints are rare. The condition is sporadic in nature and does not seem to be associated with endocrinopathy. The most frequent postoperative complaint is incomplete correction, wherein the patients are not completely satisfied with the aesthetic result (about 15% of cases). Group B streptococcal infection is associated with postpartum infections, not routine genital surgery. Scars typically heal very well, and keloid formation is not a common concern.

A 64-year-old man is brought to the emergency department after collapsing at home. Examination shows a ruptured abdominal aortic aneurysm and hemodynamic instability. The patient is taken to the operating room to undergo open repair of the aneurysm. Postoperatively, urine output decreases despite aggressive fluid resuscitation, and urinary bladder pressure is greater than 30 mmHg. Increased peak airway pressures are noted. Which of the following is the physiologic effect of increased intra-abdominal pressure in this patient? A) Direct organ compression leads to decreased systemic afterloadB) Elevation of the diaphragm leads to decreased preloadC) Elevation of the diaphragm leads to increased flow in the inferior vena cavaD) Vascular compression leads to increased flow in the inferior vena cavaE) Vascular compression leads to decreased renal vascular resistance

The correct response is Option B. Abdominal compartment syndrome (ACS) may develop rapidly after an increase in intra-abdominal pressure. Chronic causes of elevated intra-abdominal pressure (such as central obesity or large abdominal tumors) may be compensated for, but acute elevations as a result of trauma, bleeding, burn, or abdominal surgery may lead to life-threatening failure of multiple organ systems. Three mechanisms present in ACS lead to multiple organ failure: vascular compression, elevation of the diaphragm, and direct organ compression. These three forces and their interactions create a constellation of physiologic effects that lead to the circulatory collapse at the center of ACS. Vascular compression results in decreased flow to the inferior vena cava and an increase in renal vascular resistance. Diaphragmatic elevation results in decreased flow to the inferior vena cava and an increase in intrathoracic pressure and decreased cardiac pre-load. Direct organ compression leads to an increase in systemic afterload. Left untreated, these forces eventually lead to renal failure, respiratory failure, intracranial hypertension, and intestinal and hepatic ischemia.

The dominant vascular supply of the rectus abdominis muscle originates from which of the following vessels? A) Common femoralB) External iliacC) Internal iliacD) Internal mammaryE) Superficial femoral

The correct response is Option B. Component separation for closure of large abdominal wall defects was first described by Ramirez in 1990. The purpose of the surgery is to achieve abdominal wall closure with well-vascularized, innervated muscle flaps. The primary vascular supply to the rectus muscles are the deep inferior epigastric artery and vein, which arise from the external iliac vessels. The internal mammary vessels give rise to the superior epigastric arteries and veins, which is a secondary, nondominant vascular supply of the rectus muscles. The femoral vessels give rise to the superficial inferior epigastric artery and vein, which perfuse the skin and subcutaneous fat of the inferior lateral abdomen.

A 25-year-old man returns to the operating room for closure of the abdomen 2 weeks after undergoing small-bowel resection. The abdomen was left open after the resection and treated with abdominal negative pressure wound therapy. The bowel wall edema has improved, and the fascia and rectus muscles are 30 cm apart at the level of the umbilicus. There is no evidence of contamination. Abdominal wound closure is planned. There is adequate skin for primary closure. Which of the following methods is most appropriate for fascial closure? A) Acellular dermal matrix and negative pressure wound therapyB) Component separation and acellular dermal matrix interpositionC) Skin grafting and negative pressure wound therapyD) Subcutaneous tissue expansion, staged closure of skin flapsE) Submuscular tissue expansion, staged closure of fascial flaps

The correct response is Option B. Abdominal wall reconstruction after severe trauma involves evaluation of the skin and fascia. First, the surgeon needs to determine if there is sufficient skin and subcutaneous tissue for primary closure. If there is insufficient skin, then tissue expanders, local tissue rearrangement, or distant flaps need to be considered. Second, if there is insufficient fascia, then component separation with primary fascial closure and mesh onlay or a mesh interposition are options for fascial closure. Since the fascial defect is 30 cm, it is unlikely that primary fascial closure can be achieved with component separation alone. An interposition of acellular dermal matrix is appropriate to bridge the fascial gap if primary fascial closure cannot be achieved.

A 25-year-old man is scheduled to undergo soft-tissue coverage and nerve grafting using a seventh intercostal space nerve graft after he sustained a gunshot wound just above the left clavicle. On preoperative examination, he had difficulty abducting the shoulder, and injury to the brachial plexus was suspected. Between which of the following structures is the thoracic intercostal nerve located? A) External intercostal and pectoralis major musclesB) Innermost intercostal and internal intercostal musclesC) Internal and external intercostal musclesD) Parietal pleura and transverse thoracis muscleE) Transverse thoracis innermost intercostal muscles

The correct response is Option B. Although not commonly used, the thoracic intercostal nerves can provide graft material up to 12 cm in length. Harvest of the intercostal nerve has minimal donor site numbness and can be harvested through an open or endoscopic approach. The intrinsic chest muscles include (from superficial to deep) the external intercostal, internal intercostal, innermost intercostal, and transverse thoracis muscles. The external intercostal muscle is most active during inspiration. It functions to stiffen the chest wall during descent of the diaphragm to prevent paradoxical collapse of the chest. The other more internal intercostal muscles are weaker and are more involved in expiration. The intercostal nerve emerges from the spinal cord and immediately splits into a dorsal ramus (that innervates the back) and ventral ramus. The ventral ramus runs between the internal and innermost internal intercostal muscles before crossing over the internal thoracic vessels and penetrating through the intercostal muscles to supply the anterior chest skin.

A 55-year-old man who underwent abdominal surgery 10 years ago undergoes lysis of adhesions for treatment of ongoing intermittent bowel obstruction symptoms. He does not smoke cigarettes and has a history of hypertension and diabetes mellitus. A single enterotomy is made and repaired primarily. During abdominal wall closure after the intra-abdominal procedure, the fascial edges cannot be approximated without marked tension with a relaxed defect diameter maximum of 7 cm. Which of the following is the best method of repair? A) Bioprosthetic mesh bridgingB) Component separation and bioprosthetic mesh underlayC) Component separation and synthetic mesh underlayD) Component separation with no meshE) Synthetic mesh bridging

The correct response is Option B. Given the size of the defect, the patient's comorbidities, and bowel violation, the best method to optimize results is a component separation with a bioprosthetic mesh underlay. The use of bridging mesh without approximation of the fascia is not recommended due to a high recurrence rate. Instead the Ventral Hernia Working Group (VHWG) recommends reapproximation of the rectus muscle at the midline whenever possible without undue tension. This can be done by using the component separation technique originally described by Ramirez. The procedure calls for a release of the external oblique aponeurosis 1 cm lateral to the linea semilunaris, which allows for medialization of the rectus abdominis and underlying lateral musculature for primary approximation. Given the enterotomy, a bioprosthetic mesh would be recommended as opposed to a synthetic mesh, because it is likely more resistant to infection and does not necessarily need removal in a contaminated wound. It is the VHWG's preferred method to place mesh in an underlay manner, because intra-abdominal pressure pushes the mesh against the native abdominal wall instead of away from it. It also adds another layer of tissue over the prostatic material and would be preferred with a bioprosthetic because of decreased risk for bowel adhesions. Additionally, lower rates of hernia recurrence have been shown in patients who underwent component separation with mesh as opposed to those without.

A 43-year-old woman, gravida 3, para 3, undergoes a combined panniculectomy and hernia repair. The planned hernia repair is a retrorectus repair without components separation with polypropylene mesh. Which of the following best describes an advantage of using polypropylene mesh compared with a biologic scaffold? A) Decreased need for components separationB) Decreased recurrence rateC) Increased collagen cross-linkingD) Increased resistance to infection

The correct response is Option B. In general, the indication to use biologic scaffolds in hernia repairs and abdominal reconstruction is in contaminated beds. Biologic scaffolds provide an intact extracellular matrix and support tissue regeneration, and are more resistant to infection than synthetic mesh, likely because of this tissue ingrowth. Biologic scaffolds, unlike synthetic mesh, are degraded over time by collagenase. This accounts for at least some of the higher recurrence rates seen with biologics compared to synthetic mesh. Cross-linking of the scaffold may provide resistance to collagenase and improve long-term stability. Thus, better resistance to infection is a property of biologic scaffolds, as is collagen cross-linking, although the degree of cross-linking varies based on the particular scaffold. They are associated with a higher cost and an increased recurrence rate. Their high cost compared with synthetic mesh does not justify their routine use, and the recommendation for their use by the Ventral Hernia Working Group is in contaminated fields, with infected mesh and septic dehiscence. The need for components separation is based on the properties of the hernia itself and is independent of the type of mesh used. Components separation without mesh is, however, associated with a high recurrence rate.

A 33-year-old woman seeks panniculectomy to address intertrigo following a 100-lb weight loss after undergoing bariatric surgery 18 months ago. Her weight has been stable for the past 3 months, and her current BMI is 30 kg/m2. Which of the following aspects of this patient's history is most likely to interfere with insurance coverage? A) BMI of 30 kg/m2B) 3 Months of weight stabilityC) 18 Months status postbariatric surgeryD) Primary symptom of intertrigoE) 100-lb weight loss

The correct response is Option B. Many patient history factors are important when considering indications for body contouring after weight loss. Surgical indications include symptomatic rashes, large amounts of weight loss, adequate time between bariatric surgery and body contouring surgery, decreased BMI, and a substantial time period of weight stability, longer than 3 months. Additionally, insurance carriers have varying criteria to allow authorization of abdominal contouring procedures, specifically panniculectomy. Many insurance carriers require 6 months of weight stability. Severe intertrigo, 100-lb weight loss, 18 months status post bariatric surgery, and a relatively low BMI would be in keeping with frequently used clinical indications for surgery and insurance coverage criteria.

A 3-year-old child with pectus excavatum deformity is evaluated for surgical correction of the chest wall. The child has experienced mild respiratory insufficiency. Which of the following is the optimal timing of treatment for this patient? A) Surgical correction between ages 2 and 5B) Surgical correction between ages 6 and 12C) Surgical correction between ages 13 and 17D) Surgical correction at skeletal maturity

The correct response is Option B. Pectus excavatum is the most common congenital chest wall deformity, occurring in approximately 1 in 400 live births. The condition is more common in males, and there is a positive family history in 30 to 40% of patients. The etiology is thought to be multifactorial and associated with increased incidence of congenital cardiac abnormalities, connective tissue disorders (e.g., Marfan and Ehlers-Danlos syndromes), and scoliosis. Treatment options have shifted from the traditional open technique involving sternal osteotomy and resection of abnormal costal cartilage to minimally invasive options such as the Nuss procedure and minimally invasive technique for repair of excavatum (MIRPE), which utilizes thoracoscopy and placement of intrathoracic retrosternal support bars to reposition the sternum and allow gradual remodeling over a period of 2 to 4 years. The ideal timing of repair is mid-adolescence, usually between ages 6 and 12.

A 45-year-old woman comes to the office seeking consultation for an abdominoplasty. Physical examination shows diastasis recti and excess skin and fat in the infraumbilical region. Abdominoplasty with undermining of the upper abdominal flap and plication of the diastasis is planned. Decrease in overall sensation in which of the following areas is most likely after more than 3 years postoperatively? A) EpigastricB) InfraumbilicalC) Lateral abdominalD) SubxiphoidE) Suprapubic

The correct response is Option B. The area below the umbilicus and above the incision is most likely to have decreased sensation in the long term (after more than 3 years). This is true for superficial touch, superficial pain, temperature (hot and cold), and vibration. The area above the umbilicus recovers sensation to touch, pain, and vibration within 3 years; sensation to hot and cold temperatures recovers beyond 3 years. The suprapubic area has decreased sensation to temperature; however, touch, pain, and vibration recover in the short term. The areas below the xiphoid, at the lateral abdomen, as well as all other areas on the abdomen, have decreased sensation to pressure, although the infraumbilical region had the greatest change. The anterior nerve branches of the 6th to 12th intercostal nerves travel in a plane between the transversus abdominis and internal oblique muscles. They enter the internal oblique fascia, divide into two branches, and enter the posterior rectus sheath. One branch ascends 3 cm from the lateral edge of the rectus muscle to supply sensation to the skin over the lateral half of the rectus muscle. The other branch travels between the rectus and the posterior sheath before penetrating the linea alba and supplying sensation to the medial half of the rectus muscle. These nerves are divided during undermining from the abdominoplasty.

A 27-year-old woman is evaluated for a recurrent abdominal desmoid tumor. CT scan shows a mass that occupies the full-thickness right musculofascial abdominal wall, involving the rectus abdominis muscle and oblique muscles, including lateral to the semilunar line. Resection is performed. Photographs of the defect are shown. Which of the following is the most appropriate management? A) Bilateral component separation, primary skin closure with incisional topical negative pressure wound therapy, adjuvant chemotherapyB) Left component separation, bridging wide intraperitoneal underlay biologic mesh, primary closure skinC) Pedicled right anterolateral thigh flap with rectus femoris, no meshD) Placement of a bridging inlay of biologic mesh with primary split-thickness skin grafting and negative pressure wound therapyE) Placement of a bridging inlay of uncoated heavyweight polypropylene mesh with adjuvant radiation therapy

The correct response is Option B. The lesion in this patient is a recurrent desmoid tumor, also known as aggressive fibromatosis. It is a benign tumor, usually found in younger patients between 10 and 40 years of age, and is locally aggressive. It is often associated with pregnancy and previous surgery and can frequently recur. Management is en bloc, full-thickness, wide local excision (usually with frozen section confirmation of negative margins). An aggressive full-thickness abdominal wall resection is standard of care, making reconstruction more challenging. The more durable and functional reconstruction entails complete restoration of the abdominal wall, especially musculofascial components, in a primary reapproximation. However, depending on the size of the tumor and resultant defect, this may not be possible. Basic principles, after obtaining proper margins after resection of the tumor, would then be reduction in defect size to the maximal extent possible and wide bridging underlay of mesh with at least 4- to 5-cm margins in all directions. Bridging inlay, whereby a mesh is simply sewn to the margins of the defect, has clearly been shown to be inferior in terms of recurrence rates. Mesh choice can be either synthetic or biologic, although if significant contamination exists, if soft-tissue coverage is tenuous, or if one desires to decrease the amount of adhesion formation when placing mesh directly against the bowel, one should consider placement of biologic mesh, accepting the fact that there is a higher incidence of postoperative bulges using these materials, by and large. In this case, only a left component separation is possible, given that the tumor has invaded the right rectus muscle and obliques, precluding their use for myofascial advancement. There is no role for chemotherapy or radiation therapy in the treatment of these tumors. Coverage with a right anterolateral thigh flap, with or without rectus femoris, can reconstruct the soft-tissue defect, but avoiding the use of mesh in a defect over 4 cm has a significantly higher chance of a recurrent hernia and would not be standard of care. Primary skin grafting on top of a nonvascularized thick piece of acellular dermal matrix will not "take," even with use of negative pressure wound therapy.

A 65-year-old man presents with an infection of the sternum following aortic valve repair. After sternal debridement, there is a 10-cm-wide, deep wound from the clavicle to the upper abdomen. Which of the following is the most appropriate flap to reconstruct the wound? A) Latissimus dorsiB) OmentumC) Pectoralis majorD) Pectoralis minorE) Serratus

The correct response is Option B. The most appropriate flap to reconstruct the wound is omentum. Because of the large extent of the wound, the only flap listed that can adequately fill the defect and eliminate the dead space is the omentum. Pectoralis major flaps would not adequately fill the defect, particularly the inferior aspect of the wound. Pectoralis minor flaps are not used for sternal reconstruction and would not provide adequate tissue. The latissimus dorsi flap would not be able to fill the large sternal wound. Serratus flaps can be used for posterior chest wounds, but would not be able to reconstruct the large anterior chest wound.

An otherwise healthy 17-year-old girl with congenital vaginal agenesis is referred to the office for reconstruction of the vagina. Physical examination shows normal external genitalia but an absent vaginal canal. She desires a single-stage procedure with minimal scarring and maximal sensation. Which of the following is the most appropriate reconstructive option for this patient? A ) Bilateral gracilis musculocutaneous flapsB ) Bilateral pudendal fasciocutaneous flapsC ) Extended groin flapD ) Split-thickness skin graft and mold (McIndoe procedure)E ) Vertical rectus abdominis musculocutaneous flap

The correct response is Option B. The most appropriate management is reconstruction with bilateral pudendal fasciocutaneous flaps. These flaps are based on the branches of the superficial perineal vessels, which are the terminal branches of the internal pudendal vessels. The course of the superficial perineal nerve follows these vessels; therefore, this flap can be raised as a sensate flap. These are relatively simple flaps to raise, and they are reliable and completed in one stage. They are thin, sensate flaps and are ideally suited for vaginal reconstruction. The donor site can be closed primarily and the scar is well hidden within the inner groin crease. The McIndoe procedure involves a split-thickness skin graft reconstruction of the vagina using a mold, followed by the continued use of a vaginal dilator to prevent constriction. The skin graft donor site scar is not as inconspicuous as a linear scar in the groin crease. Extended groin flaps have been described for vaginal reconstruction; however, these flaps are insensate. Gracilis and rectus abdominis musculocutaneous flaps are workhorse flaps for vaginal and pelvic reconstruction. However, they too are insensate and have large, conspicuous donor site scars.

A 1-day-old female newborn is evaluated because of repair of a lumbar myelomeningocele. After dural repair, physical examination shows the spinal cord at the base of the wound with a 4 × 4-cm soft-tissue and skin defect. Which of the following is most appropriate to reconstruct the wound? A) Gluteal muscle flap and skin advancement flapB) Paraspinous musculofascial flap and skin advancement flapC) Skin advancement flap onlyD) Split-thickness skin grafting

The correct response is Option B. The most appropriate method to reconstruct the wound is a local musculofascial flap and skin advancement flap. The major principle of myelomeningocele repair is to obtain a well-vascularized layer of soft-tissue coverage between the dural and skin repairs. The fascia overlying the paraspinous muscles can be turned over as flaps, followed by paraspinous muscle advancement flaps to cover the underlying dural repair. This vascularized soft-tissue layer will minimize the risk of cerebrospinal fluid contact with cutaneous bacteria and subsequent meningitis if either the dural repair or skin repair breaks down. A split-thickness skin graft over the dura would not adequately protect the spinal cord. Closing the skin directly over the dural repair using skin advancement flaps would place the child at risk for meningitis in the event of a cerebrospinal fluid leak or if wound breakdown occurred along the incision line of the widely undermined skin flaps. The use of a regional gluteal or latissimus muscle flap to cover the dural repair is unnecessary because local tissue (paraspinous muscles and fascia) is available. Harvesting the gluteal or latissimus muscles also may cause significant donor site morbidity in a child already at risk for ambulatory problems from a neurologic deficit.

A 62-year-old woman presents with a new-onset draining sinus of the left thoracic cage with associated indurated skin. Medical history includes bilateral breast cancer that was managed with bilateral radical mastectomy with radiation therapy 27 years ago. On CT scan, the image (shown) is consistent with osteoradionecrosis. Resection of affected skin, soft tissue, and thoracic cage produces a 35 × 20-cm soft-tissue defect and a skeletal defect spanning five ribs. A photograph of the defect is shown. The thoracic cage is fibrotic and noncompliant because of previous radiation. Which of the following approaches is most appropriate for reconstruction? A) Free omental flap with skin graft over titanium mesh and reconstruction platesB) Left latissimus dorsi muscle flap with skin graft over acellular dermal matrixC) Left rectus abdominis turnover flap with skin graft over methyl methacrylate sandwichD) Reverse abdominoplasty advancement over ePTFE patchE) Right pectoralis muscle turnover flap over polypropylene mesh

The correct response is Option B. The most appropriate option for this patient is a left latissimus dorsi muscle flap with skin graft over acellular dermal matrix, given the alternatives listed. Basic principles of thoracic reconstruction include: debridement of devitalized tissue, removal of foreign bodies, establishment of healthy wound bed, restoration of stability/structure (generally reconstruction of skeleton if more than four ribs or a greater than 5-cm-diameter defect is involved), restoration of normal respiratory mechanics, protection of vital structures/organs, obliteration of dead space, provision of durable coverage, and delivery of an aesthetic result. However, if a patient has been previously irradiated, and therefore loses compliance of the thoracic cage because of radiation-induced fibrosis, skeletal reconstruction may not be mandatory if there is no paradoxical motion of the thoracic cage upon respirations and there is preservation of respiratory efficiency. Such is the case with this patient. A left rectus turnover flap would not be a good option for two reasons: 1) as can be seen in the image, the left internal mammary artery has been harvested, thereby compromising the superior epigastric vessel on which this flap would be based, and 2) it is insufficient to provide enough soft-tissue coverage of a defect this size. Furthermore, as indicated above, methyl methacrylate would not be mandatory in this patient. A right pectoralis turnover flap is insufficient to cover a defect this size. A free omental flap can be used to reconstruct this defect (as can a pedicled omental flap), but again, thoracic skeletal reconstruction would not be mandatory in this previously irradiated patient; furthermore, even if it were, titanium mesh and reconstruction plates would not be utilized. A reverse abdominoplasty flap (Ryan procedure) would not be able to cover a defect this size.

To maintain innervated muscle flaps during abdominal component separation, which of the following is the most appropriate plane of dissection? A) Below subcutaneous fat pad, above fasciaB) Below fascia, above external obliqueC) Below external oblique, above internal obliqueD) Below internal oblique, above transversalisE) Below transversalis, above peritoneum

The correct response is Option C. Component separation for closure of large abdominal wall defects was first described by Ramirez in 1990. The purpose of the surgery is to achieve abdominal wall closure with well-vascularized, innervated muscle flaps. The muscles of the abdominal wall are innervated by the intercostal nerves from T7-L4. The plane below the external oblique and above the internal oblique is an avascular plane that will allow for medial advancement of the rectus muscle flaps while protecting the intercostal nerves that run under the internal oblique.

A 46-year-old man presents with a midline 18-cm-wide ventral hernia 1 year after undergoing midline exploratory laparotomy for a bowel resection and right end ileostomy. Medical history includes significant weight loss through diet and exercise. His weight has been stable for 2 years. BMI is 29 kg/m2. He undergoes bilateral component separation with biologic mesh bridged between the rectus muscles and concomitant panniculectomy. Which of the following clinical characteristics will most likely increase the likelihood of hernia recurrence? A) BMI greater than 24.9 kg/m2B) Bridged biologic mesh hernia closureC) Concomitant panniculectomyD) Presence of an end ileostomyE) Prior abdominal surgery

The correct response is Option B. The patient presents after significant weight loss with a wide midline ventral hernia, right end ileostomy through his rectus muscle, and an abdominal pannus. Given the 18-cm waist of the hernia defect, he is being counseled that only a bridged repair with a biologic mesh will be possible rather than total muscular coverage for the midline defect. Hernia recurrence is a major problem for patients and can be associated with specific characteristics. When the technique of bilateral component separation and inlay biologic mesh repair is being performed, the most important predictor of recurrence is whether the rectus muscle and fascia will be able to be closed at midline, creating a total submuscular repair, or whether the mesh will be bridged. A bridged repair is associated with a 33% chance of recurrence at 3 years compared to 6.2% for total muscle coverage with fascial closure at midline. With a BMI of 29 kg/m2, the patient remains overweight despite his prior stable weight loss. Surgical site occurrences are increased in the overweight patient with a 26.4% incidence versus 14.9% in patients with BMI less than 24.9 kg/m2. Similarly, skin dehiscence is significantly increased in the overweight patient (19.3% versus 7.2%), while hernia recurrence rates are not statistically significant (11.4% versus 7.7%). Concomitant panniculectomy was associated with an increase in surgical site occurrences and skin dehiscence, but hernia recurrence rates were not affected. Similarly, patients with existing ileostomies or stomas complicated by parastomal hernias do have a significantly increased surgical site occurrence rate (34.1% with parastomal and midline hernia versus 18.7% with midline hernia only) but hernia recurrence rates are not affected. Prior abdominal surgery will be in the clinical history of all incisional hernia patients.

A 30-year-old woman is evaluated for a mass on the anterior abdominal wall that has been growing over the past several months. Imaging and examination of the specimen obtained on biopsy confirm a diagnosis of desmoid-type fibromatosis arising from the anterior abdominal musculature. Which of the following is the most accurate statement regarding this condition? A) It is associated with previous radiation exposureB) It is locally aggressive with remote risk of metastatic diseaseC) It usually arises from an untreated lipomaD) Primary treatment is combination chemotherapy/radiationE) There is an autosomal recessive genetic inheritance pattern

The correct response is Option B. This patient has a desmoid tumor, a rare, locally infiltrative mesenchymal neoplasm that is most commonly found in adolescents and young adults. It does not metastasize but can have a very unpredictable and aggressive natural history. Treatment can range from close observation (as some tumors will self-involute) to radical excision with wide margins. Traditionally, surgical excision can be difficult because of the high risk of local recurrence. Desmoid tumors are not associated with chemical or radiation exposures, or lipomas. Metastatic disease is not a characteristic of desmoid tumors. Although treatment modalities for desmoids are evolving, they are not treated with chemotherapy/radiation therapy as a primary modality. Primary treatment is frequently observation for small or slowly changing tumors. For rapidly growing tumors, radical excision is frequently performed. There does not appear to be a genetically inherited component for desmoid tumors, although they are associated with random mutations of the ?-catenin (CTNNB1) and adenomatous polyposis coli (APC) genes.

A 31-year-old man who recently had a 100-lb (45.4-kg) weight loss presents with a 5-year history of bilateral gynecomastia. Results of endocrine workup are negative, and testicular ultrasonography shows no abnormalities. He does not take any hormones. Physical examination shows an estimated 300 g of primarily glandular tissue and marked skin redundancy. Which of the following is the best treatment for the degree of gynecomastia in this patient? A) Direct gland and skin excision with liposuctionB) Direct gland and skin excision with nipple transpositionC) Direct gland excision and cryolipolysisD) Liposuction and direct gland excision onlyE) Liposuction and nipple transposition

The correct response is Option B. This patient has class IIAii gynecomastia per the McMaster classification of gynecomastia. To treat this degree of gynecomastia, in which there is 250 to 500 g of tissue that is primarily glandular and associated with marked skin redundancy, direct gland and skin excision with nipple transposition (with or without liposuction) is indicated. Skin excision patterns include a boomerang pattern to correct vertical and horizontal skin excess primarily in patients with massive weight loss. Transverse incisions are used to remove vertical excess. Liposuction would not address the glandular tissue, and skin excision is needed to treat the skin redundancy. Liposuction combined with direct gland excision also does not correct the skin redundancy. Cryolipolysis is indicated for fat reduction, which would not treat the primarily glandular component. Direct gland and skin excision with liposuction does not correct the nipple position.

An 18-year-old woman is evaluated for chest wall and breast asymmetry. She has a history of pectus excavatum and underwent surgery as a child for placement of a correction bar, which was subsequently removed 3 years after insertion. Since the removal of the bar, she has noticed progressive recurrence of her chest wall deformity with associated breast asymmetry. Physical examination shows a thin patient with a 4-cm deep concavity involving the lower end of the sternum and medial distortion of the right breast. The patient denies shortness of breath or chest pain. Which of the following treatment options is the most appropriate recommendation for contour improvement? A) Autologous transfer of fat tissue to the sternal defect and right breastB) Implantation of a customized silicone elastomer device with concurrent augmentation mammaplastyC) Injection transplantation of cultured autologous chondrocytesD) Placement of bilateral silicone breast implants greater than 550 ccE) Reinsertion of the correction bar with sternal wiring

The correct response is Option B. When considering how best to treat pectus excavatum in the female patient, it is important to recognize that the majority of the contour deformity is due to the thoracic concavity with only a small portion of the deformity due to actual breast tissue hypoplasia. However, reinsertion of a correction bar is not always successful in correcting the thoracic deformity in post-adolescent patients whose bones have ossified, and is only done in extreme cases. Augmentation mammaplasty is also unsuccessful, regardless of implant volume, since, as stated above, only a small portion of the defect is actually caused by breast tissue asymmetry. Although some might consider autologous fat grafting a viable option for correcting both the chest wall defect and the hypoplastic breast, there have been reports that indicate that injecting between the tight adhesions of presternal skin and bone is technically challenging and rarely successful. Injection of cultured autologous chondrocytes, which is still in the early stages of research and development, would prove to be equally technically challenging, and at this time, de novo generation of cartilage or fat for injection into soft-tissue defects is still unproven. As a result, customized silicone elastomer implants are commonly used in conjunction with augmentation mammaplasty to provide consistent and reliable correction of pectus excavatum and breast asymmetry in the female patient. Studies have shown that placing a custom sternal prosthesis and bilateral breast implants during one surgery is both safe and effective in producing an improved aesthetic result.

An 18-year-old woman comes to the office to discuss her congenital breast asymmetry. Physical examination shows asymmetry in the size and shape of the right breast and nipple-areola complex compared with the left breast. Additionally, there is an absence of the right anterior axillary fold. A photograph is shown. Which of the following is the most likely origin of the absent muscle in this patient? A) EctodermB) EndodermC) MesodermD) NeuroectodermE) Notochord

The correct response is Option C. The patient above has Poland syndrome. Poland syndrome represents a spectrum of congenital chest wall anomalies ranging from the simple form (depicted in the preoperative photograph) to complex. The pathognomonic feature is the absence of the sternocostal head of the pectoralis major muscle. A variety of other ipsilateral chest wall and upper extremity malformations may be present, including absence of ribs two through five, foreshortening of the limb, brachysyndactyly, hypoplasia/aplasia of the breast and nipple-areola complex, and absence/hypotrophy of various trunk muscles including the latissimus dorsi, serratus anterior and external oblique. Although a few familial cases have been reported, Poland syndrome is believed to be sporadic in nature with an incidence of 1:100,000. There is a male-to-female predilection of 3:1, with a right-sided predominance in boys of 2:1. The patient is shown two months after reconstruction with a right latissimus flap and bilateral silicone gel-filled implants. The trunk develops from the fusion of the ectoderm/neural crest layer with the somatic mesoderm during the fourth week of gestation. Mesoderm gives rise to somites, which differentiate into the dorsolateral dermomyotome and the ventromedial sclerotome. The dermomyotome gives rise to the musculature of the trunk and extremities. The sclerotome surrounds the developing spinal cord and notochord and forms the vertebral bodies and spinal skeletal structure. The notochord acts as a pathway for the development of the spinal cord and vertebral bodies. The notochord degenerates as the vertebral bodies develop and becomes the nucleus pulposus of the intervertebral disks. The exact pathophysiology of Poland syndrome has not been elucidated. One theory suggests a mechanical disruption of the embryonic blood supply in the subclavian/vertebral systems during the sixth to seventh week of gestation. Another theory suggests the etiology to be an injury or developmental failure of the mesodermal plate during the third to fourth week of gestation. Endoderm is the primitive germ layer that gives rise to the epithelium of the respiratory and gastrointestinal tracts. Ectoderm is divided into surface ectoderm and neuroectoderm. Surface ectoderm gives rise to the epidermis, adnexal structures of the skin, and the mammary glands. Neuroectoderm becomes the central nervous system, various neural ganglia, and the branchial arches.

QUESTION A) Advancement of the vertical rectus abdominis musculocutaneous flapsB) Release and advancement of the external oblique musclesC) Release and advancement of the rectus, internal oblique, and transversus abdominis musclesD) Rotation of the anteriolateral thigh flapsE) Rotation of the transverse rectus abdominis myocutaneous flaps

The correct response is Option C. A component separation procedure allows for the primary closure of central abdominal wall musculofascial defects without excessive tension. In the classically described procedure, the external oblique aponeurosis is released lateral to the linea semilunaris, allowing for medial advancement of the rectus abdominis along with the internal oblique and transversus muscles. The innervation and vasculature to these structures are preserved in order to maintain functional integrity of these abdominal wall muscles. The vertical and transverse rectus abdominus musculocutaneous flaps along with the anterolateral thigh flaps are primarily used to reconstruct soft-tissue defects but are unlikely to result in a functional abdominal wall.

A 54-year-old man presents for elective repair of a large, recurrent ventral hernia. Plastic surgery is consulted because of the loss of abdominal domain. Posterior component separation with placement of biologic mesh is planned. Above the level of the arcuate line, which of the following structures come together to form the posterior rectus sheath? A) External oblique aponeurosis and internal oblique aponeurosisB) External oblique aponeurosis, internal oblique aponeurosis, and transversus abdominis aponeurosisC) Internal oblique aponeurosis and transversus abdominis aponeurosisD) Transversalis fascia and peritoneum

The correct response is Option C. A comprehensive understanding of the anatomy of the anterior abdominal wall is critical when performing posterior component separation. Above the arcuate line, the anterior rectus sheath is derived from the external oblique aponeurosis and the anterior component of the internal oblique aponeurosis. The posterior rectus sheath is derived from the posterior component of the internal oblique aponeurosis and the aponeurosis of the transversus abdominis muscle. The rectus abdominis muscle sits between the anterior and posterior rectus sheaths superior to the arcuate line. Below the arcuate line, the anterior rectus sheath is derived from the aponeuroses of all three muscles: the external oblique, the internal oblique, and the transversus abdominis muscles. The posterior rectus sheath does not exist below the arcuate line. The rectus muscle is only separated from the abdominal viscera by the transversalis fascia and the peritoneum.

A 55-year-old obese woman presents with sternal wound dehiscence 10 days after undergoing pectoralis major flap reconstruction. Medical history includes stable hypertension, coronary artery disease, and coronary artery bypass grafting complicated by mediastinitis. Which of the following factors is a predictor for recurrent sternal wound dehiscence in this patient? A) AgeB) Coronary artery diseaseC) Female sexD) HypertensionE) Pectoralis major flap harvest

The correct response is Option C. A retrospective chart review of 77 patients was conducted over a 7-year period, focusing on patients with sternum dehiscence who underwent pectoralis major transposition for sternal reconstruction. Female sex, smoking, detachment of the humeral insertion, and operation time are associated with postoperative wound complications. Breast size in women was previously found to be a risk factor for the development of post-sternotomy mediastinitis. Jones et al. found that recurrent dehiscence after tissue-flap coverage of the sternum occurs more frequently in obese women with large pendulous breasts. An association between female sex and recurrent wound dehiscence was also found in the study. This might have been caused by traction of large breasts on the wound edges or devascularization of mammary tissue. Due to the retrospective nature of this study, breast size as a cofactor could not be included in the analysis. Age, pectoralis major harvest, hypertension, and coronary artery disease are not independent risk factors for recurrent sternotomy dehiscence.

The postoperative CT scan shown is obtained to evaluate a wound dehiscence in a patient who underwent left-sided unilateral reduction mammaplasty for asymmetry six weeks ago. Which of the following upper extremity deformities is most likely to be found in this patient? A) Contralateral radial club handB) Contralateral "pouce flottant" thumbC) Ipsilateral brachysyndactylyD) Ipsilateral radial head subluxationE) Ipsilateral type IV Wassel preaxial polydactyly

The correct response is Option C. Assessment of the chest CT shows right-sided absence of the pectoralis major and minor muscles and breast hypoplasia. The patient suffers from Poland syndrome, which is a congenital disorder of unknown etiology with the prevailing theory being hypoplasia of the subclavian artery or its branches during the sixth week of embryogenesis. Variability exists in physical findings with the most common being: anterior axillary fold and pectoralis major sternal head absence, breast gland thinning, rib and cartilage hypoplasia, and ipsilateral brachysyndactyly. After local wound care and antibiotic therapy, the patient had resolution of her symptoms. A type IV Wassel preaxial polydactyly is the most common congenital thumb duplication but is not associated with Poland syndrome. Radial club hand is more common than ulnar club hand, but has no association with Poland syndrome. Both are congenital hand deformities, but unrelated to the pathological condition mentioned. Radial head subluxation is also known as "nursemaid's elbow." Nursemaid's elbow is a common injury of early childhood. It is sometimes referred to as "pulled elbow" because it occurs when a child's elbow is pulled and partially dislocates. There is no connection between Poland syndrome and increased incidence of radial head subluxation. Type IV Manske modification of the Blauth classification thumbs (the "pouce flottant" thumb) have rudimentary elements and are attached to the hand by a small skin bridge. These thumb anomalies are not associated with Poland syndrome.

A 42-year-old man is brought to the emergency department after sustaining a traumatic, sharp amputation of the penis at the level of the proximal shaft during a domestic abuse incident. Repair of which of the following structures is necessary to obtain successful replantation of the penis? A ) Cavernous arteryB ) Circumflex arteryC ) Deep dorsal arteryD ) Pudendal arteryE ) Subtunical venous plexus

The correct response is Option C. Before the advent of microsurgical techniques, the procedure to replant a penis was rudimentary - surgeons would simply suture the approximate corporal bodies and the overlying tunica. Although successful occasionally, this technique was prone to partial necrosis of the glans or shaft and was associated with sexual and erectile dysfunction. The use of microsurgical techniques has allowed surgeons to repair transected structures more precisely and achieve significantly improved results. A review of the current published literature on penile replantations finds that, in addition to repair of the urethra, the three neurovascular structures that are repaired consistently are the dorsal nerves, dorsal veins, and the deep dorsal artery. Generally, a suprapubic catheter is placed to divert urinary flow for 2 to 3 weeks while the urethral repair heals. The cavernous artery is buried within the substance of the corpus cavernosum and is technically difficult to repair. The circumflex artery runs circumferentially through collateral circulation and would not be repaired normally. The pudendal artery is a branch of the internal iliac vessels and is located much more proximally in the pelvis. It gives rise to the inferior rectal artery, perineal artery, posterior scrotal artery, and the deep dorsal artery. The subtunical venous plexus is extremely small and would be technically difficult to repair.

A 59-year-old man is scheduled for reconstruction of a central abdominal wall hernia measuring 10 cm wide x 30 cm long. A surgical approach using posterior component separation is planned. Which of the following locations for fascial incision most accurately describes the technical considerations of this procedure? A) Across the lateral intercostal neurovascular bundlesB) Along the mid-axillary lineC) Medial to the linea semilunarisD) Parallel to the subcostal borderE) Vertical bisection of the rectus muscles

The correct response is Option C. Component separation of the abdominal wall was initially described for the anterior components, that is, those which are located anterior to the rectus muscle. A more recent development involves component separation of the layers located posterior to the rectus fascia. The technique of posterior component separation begins with a vertical incision of the posterior rectus sheath 0.5 cm medial to the linea semilunaris and continues laterally in the avascular plane posterior to the transversalis muscle. It can extend as far posteriorly as the psoas muscle if needed. In the event that the posterior layer cannot be approximated in the midline, an interposition patch of omental fat, hernia sac, or absorbable mesh is used. Concurrent use of non-absorbable mesh to reconstruct deficient anterior layers may be used in conjunction with the posterior separation technique as long as the mesh is separated from viscera with an intact posterior layer. A benefit of the posterior separation technique is the preservation of the lateral neurovascular bundles preserving the dynamic function of the rectus muscles. The technique may be utilized even when fascial defects are not situated in the midline or are located adjacent to bony landmarks.

A 55-year-old man presents with ventral incisional hernia of the abdomen. BMI is 32 kg/m2. Medical history includes an exploratory laparotomy following a traumatic injury 1 year ago. The abdominal fascia was closed primarily at the end of the procedure. Physical examination shows a fascial deficit 10 cm in width. CT scan demonstrates that rectus muscles are intact bilaterally and 12 cm apart in the periumbilical region. Hernia repair is performed, but the fascia cannot be brought together primarily at the midline. Which of the following operations is most likely to result in the lowest risk for future hernia formation in this patient? A) Bilateral component separationB) Bilateral component separation with onlay mesh reconstructionC) Bilateral component separation with underlay mesh reconstructionD) Inlay mesh reconstructionE) Unilateral component separation

The correct response is Option C. Long-term hernia risk is lowest following primary fascial closure and placement of mesh, either in the retrorectus position or as an underlay. Inlay mesh reconstruction, in which primary fascial closure is not possible and the mesh serves as a bridge, is associated with the highest rates of abdominal hernia formation. Unilateral or bilateral component separation may allow for primary fascial closure, but placement of a mesh augments the repair and reduces hernia recurrence. Regarding onlay mesh placement, in which the mesh is placed superficial to the fascia, hernia recurrence rates have been shown to be higher in obese patients than when the mesh is placed as an underlay.

Which of the following surgical options is most important for gynecomastia patients with significant glandular hypertrophy? A) Areolar reductionB) Excess skin removalC) Lateral lipectomyD) Subtotal glandular resectionE) Ultrasonic liposuction

The correct response is Option D. In patients with a significant glandular component, it is important to do a subtotal glandular resection to prevent recurrence and provide the best chance at adequate contour. Ultrasonic liposuction has been used to attempt subtotal removal of gland tissue, but it does not reliably produce a subtotal resection. Areolar reduction is usually not necessary due to the contractile nature of the tissue. Skin resection may be necessary, most commonly in the massive weight loss patient, but does not necessarily play a role in the glandular component of the resection. Lateral suction lipectomy does not correct glandular hypertrophy.

A newborn is noted to have a lesion of the midline of the lower back consisting of a protruding membrane which covers meninges, cerebrospinal fluid (CSF), and neural structures. Which of the following is the primary goal of surgical repair? A) Hydrocephalus mitigationB) Increase in lower extremity strengthC) Infection preventionD) Prevention of tethered cord syndromeE) Restoration of bowel or bladder function

The correct response is Option C. Meningomyelocele is the most common neural tube defect. It involves dorsal herniation of the meninges and spinal cord through the vertebrae and may produce motor and sensory nerve deficits. It is often diagnosed prenatally by elevated maternal serum alpha fetoprotein and ultrasonography. Treatment of larger defects often involves both neurosurgery and plastic surgery teams. After repair of the neural placode, the goals of soft tissue reconstruction are to cover and protect the neural element, prevent infection, and avoid any cerebrospinal fluid leak. Ideally this is performed within the first 24 to 48 hours of life. Larger defects are often best reconstructed with muscle flaps, fasciocutaneous flaps, or a combination of both. Many different flaps have been described, but considerations for adequate vascularity (such as inclusion of perforator blood vessels within geometrically designed flaps) and closure without tension are paramount. While hydrocephalus is a common finding in patients with meningomyelocele, it is treated with cerebrospinal fluid shunting if required. Meningomyelocele repair does not regain or improve neural abilities that are not present at birth, such as bowel and bladder function, and lower extremity motor and sensory function. Symptoms related to tethering of the spinal cord may develop as the patient grows in as many as 20 to 50% of children who undergo meningomyelocele repair shortly after birth and many may require surgery to release the scar tissue attached to the cord. However, this condition is not prevented by meningomyelocele repair.

A newborn male infant who is born at 36 weeks' gestation via cesarean delivery has a large defect of the anterior abdominal wall. Examination shows matted bowel loops coming through the defect lateral to the umbilical cord. No other abnormalities are noted. Which of the following associated findings is/are most likely? A) Abnormal karyotypeB) Constriction rings with limb and digital amputationsC) Elevated maternal serum alpha fetoprotein (MSAFP)D) Hypoglycemia, macrosomia, and macroglossiaE) Translucent membrane covering bowel

The correct response is Option C. Omphalocele (OC) and gastroschisis (GS) represent the two most common congenital abdominal wall defects, with a prevalence of approximately 3 to 4 per 10,000 live births/fetal deaths/stillbirths/pregnancy terminations each. Precise pathoetiologies are unclear, but developmental pathways and characteristics at the time of birth are notably distinct. OC is characteristically a midline partial-thickness abdominal wall defect covered by a membrane of amnion and peritoneum occurring within the umbilical ring and containing abdominal contents. GS is characteristically a full-thickness, paraumbilical abdominal wall defect associated with eviscerated bowel. Both OC and GS are associated with elevated maternal serum alpha fetoprotein (MSAFP). For comparison, MSAFP values average twice that recorded in pregnancies with open spina bifida, and similar to values recorded with anencephaly. An elevated MSAFP is an indication for thorough ultrasound examination of the fetus for anatomical abnormalities. Multiple chromosomal abnormalities have been associated with at least 60% OC cases, including trisomy -18, -13, -21, Turner syndrome, and triploidy. By contrast, GS is associated with abnormal karyotype in about 1% of cases, usually in the setting of other congenital abnormalities. The definite treatment of both OC and GS is surgical once optimal resuscitation is achieved. Primary closure is associated with better survival rates if it can be achieved without compromise of intestinal blood flow or other hemodynamic or respiratory embarrassment. Large defects are frequently managed with temporary abdominal silos which are gradually reduced over the course of days to weeks in a form of visceral tissue expansion followed by delayed abdominal wall closure. The long-term outcome in isolated cases of OC and GS are generally good, although they can be associated with gut motility impairment, gastroesophageal reflux, ventral hernias, and late obstructive episodes. Constriction rings with limb and digital amputations are found in amniotic band sequence but are not characteristic of OC or GS. GS is not characteristically associated with hypoglycemia, macrosomia, or macroglossia.

A 21-year-old woman comes to the office because of a lump 2 cm below the inframammary fold. She says she has "always had it," but it grew larger during pregnancy and has not decreased. The mass was painful during breast-feeding, and it is occasionally tender. Which of the following is the most likely diagnosis? A) Epidermal inclusion cystB) LipomaC) PolymastiaD) PolytheliaE) Sarcoma

The correct response is Option C. Patients often come to plastic surgeons with subcutaneous masses. An understanding of the differential diagnosis is helpful to counsel patients. Accessory breast tissue (polymastia) along the milk line is common. This breast tissue is responsive to hormonal influence, and patients will describe cyclical pain and swelling coinciding with menses and with milk letdown. Surgical excision can be performed. Polythelia is the presence of accessory nipples. Lipomas, epidermal inclusion cysts, and sarcomas can occur as subcutaneous masses, but they do not change in character based on hormonal influence.

An 18-year-old woman with right-sided Poland syndrome requests improvement in the appearance of her chest. Physical examination shows absence of an anterior axillary fold on the right side and a pectus excavatum deformity with an overlying hypoplastic right breast. The nipple-areola complex is small, lateral, and raised by about 3 cm compared with the left side. Which of the following thorax, breast, nipple-areola complex (TBN) classifications best characterizes this patient's Poland syndrome deformity? A) T1, B1, N1B) T1, B2, N1C) T2, B1, N2D) T3, B2, N3

The correct response is Option C. Poland syndrome is likely a multifactorial genetic syndrome related to the embryologic timing and development of the subclavian arch. Its unifying finding is absence of the pectoralis major muscle but is variable in other manifestations of underdevelopment of the chest wall, breast, and ipsilateral upper extremity. Poland syndrome is most often reported as more frequent in males, but some series show equal expression in males and females. Additionally, up to 10% of patients may have associated dextrocardia. Having a way to describe or classify a deformation in an organized fashion is helpful in planning reconstruction, determining results and outcomes, and discussing cases with colleagues. The thorax, breast, nipple-areola complex (TBN) system was proposed and published by Romanini et al. to do just that. Since that publication, further research by the group based on the TBN classification has been published and others have suggested modifications to include the presence or absence of ipsilateral upper extremity anomalies. ThoraxT1: absence of all or part of pectoralisT2: T1 + pectus excavatum or carinatumT3: T1 + rib aplasia (usually 3 and 4)T4: T1 + T2 + T3BreastB1: hypoplastic breastB2: breast aplasia (amastia)Nipple-areola complexN1: hypoplastic NAC less than 2 cm displacedN2: hypoplastic NAC more than 2 cm displacedN3: atheliaThe patient in the scenario is best described by the TNB classification as T2 (no pectoralis, pectus excavatum), B1 (breast hypoplasia) and N2 (hypoplastic NAC greater than 2 cm displaced). According to this classification, she is probably best served by correction of the pectus first, then correction of the breast.

A 4-year-old boy is scheduled to undergo abdominal wall repair using a component separation technique. He underwent living donor liver transplantation during infancy, at which time the graft was noted to be too large for size. A midline incisional hernia was intentionally made to facilitate organ function after transplantation. Current physical examination shows ventral herniation with marked loss of domain. During abdominal wall repair by component separation, parallel aponeurotomy is most likely to be performed immediately lateral to which of the following anatomical structures? A ) Arcuate line of DouglasB ) Inguinal ligamentC ) Linea semilunarisD ) MidlineE ) Tendinous inscription

The correct response is Option C. Separation of components of the abdominal wall is fast becoming a first-choice procedure for large ventral defects. The procedure involves medial advancement of the rectus abdominis muscle flap after division of the aponeurosis or fascia of the external oblique muscle layer. Properly identifying the correct fascia to incise is a critical step in the procedure, and it relies on knowledge of the abdominal wall anatomy. The rectus abdominis muscles are separated by the midline linea alba, and each rectus muscle is medial to the layered triad of muscles—the external oblique, internal oblique, and transversus. Division of external oblique fascia is properly performed lateral to the linea semilunaris, which represents the lateral border of the rectus sheath, which is formed by the contributions of the external oblique, internal oblique, and transversus layers as the envelope of the rectus abdominis muscle. Above the arcuate line, the later layers split around the rectus muscle at the level of the internal oblique, forming both the anterior and posterior rectus sheaths; however, below the arcuate line, all layers travel anteriorly, forming the anterior sheath but no distinct posterior rectus sheath. The inguinal ligament exists between the anterior superior iliac spine and the pubic symphysis and should not be divided in component separation. Transverse tendinous inscriptions interrupt the rectus sheath, and division of the anterior rectus sheath or the inscriptions is not part of the classic component separation.

An otherwise healthy 27-year-old man comes to the office for removal of asymptomatic enlarged breasts that have persisted unchanged since onset at age 14. He takes no medications and does not use recreational drugs. Physical examination shows symmetrical collections of rubbery, firm subareolar tissue 4.5 cm in diameter. There is scant surrounding fatty tissue and no ptosis; areolas are 28 mm in diameter. Which of the following is the most appropriate management of this patient's condition? A) Circumferential periareolar resection with liposuctionB) Mastectomy via Wise pattern incisionC) Observation to allow for involutionD) Oral hormone-blocking medicationE) Subareolar tissue ressection via pull-through excision technique

The correct response is Option E. Gynecomastia, enlargement of the male breast, may occur as physiologic temporary overgrowth of the adolescent breast or it may appear during adulthood as a result of numerous etiologies. It can consist of various proportions of excess subareolar fibrous breast tissue and adipose tissue in the periphery, with the extent often depending upon the habitus of the individual. Appearing at about age 14 in more than 65% of healthy boys, gynecomastia will typically resolve within 2 years of onset in otherwise healthy males. Persistence of adolescent-onset gynecomastia after age 21 is unlikely to resolve with conservative measures. Hormone blockers have no place in the management of persistent breast tissue in otherwise healthy individuals. A wide variety of surgical treatment options for resection of redundant male breast tissue have been published. These include direct excision through a number of described incisions, traditional and ultrasound-assisted liposuction, and recently, use of an arthoscopic shaver. Ultrasound-assisted liposuction may have an advantage in stimulating skin retraction for cases where mild to moderate ptosis exists when scar minimization is desired. For otherwise uncomplicated gynecomastia, excision of fragmented subareolar tissue via a limited areolar border incision with the pull-through technique is the most appropriate choice of therapy. When redundant skin exists in addition to excess male breast tissue, immediate skin resection via periareolar, transverse lenticular, omega-shaped, or Wise (inferior pedicle) incision are among the numerous methods described. Regardless of the resection method, an evolving trend favors allowing skin to retract for 6 to 9 months before determining whether there is a need to subject the individual to additional incisional scarring.

A 40-year-old man undergoes ventral hernia repair with biologic mesh and fascial closure at the midline. A bilateral component separation technique with incision of the external oblique fascia and muscle lateral to the linea semilunaris and dissection in the plane between the external and internal oblique muscles, and separation of the rectus muscle off of the posterior rectus fascia is performed. At which of the following levels can the least amount of advancement of the medial fascial edges be expected? A) Midway between the umbilicus and pubisB) Midway between the umbilicus and subcostal marginC) Subcostal marginD) SuprapubicE) Umbilicus

The correct response is Option C. The component separation technique can be used to achieve medial transposition of the rectus muscle and overlying anterior fascia. The surgery involves division of the external oblique fascia and muscle lateral and parallel to the linea semilunaris. The plane deep to the external oblique muscle, which is relatively avascular, is then dissected laterally. The rectus muscle is also separated off of the posterior rectus sheath (using access from the medial laparotomy or hernia incision). This allows for medial advancement of the rectus muscle, overlying anterior rectus sheath, internal oblique muscle, and transversus muscle as a unit. The segmental neurovascular bundles course deep to the internal oblique muscle and penetrate into the rectus muscle 10 to 25 mm medial to its lateral margin. The component separation technique, when performed in the scenario described, can give unilateral advancement toward the midline approximately 10 cm at the level of the umbilicus, which equates to a bilateral advancement of 20 cm. The least amount of advancement is in the subxiphoid and subcostal regions, often making more cranially located defects more difficult to close. Since it was originally reported in 1990, several modifications and variations of this technique have been described in the literature. These include perforator-preserving and/or endoscopic techniques to methods that describe additional maneuvers to increase mobilization or improve durability with the addition of biologic or prosthetic meshes.

A 50-year-old woman with sarcoma of the right lateral chest wall has completed neoadjuvant radiation. Surgical resection is performed and results in a 6-cm diameter skeletal defect of the chest wall. Soft-tissue reconstruction only is planned. Soft-tissue only reconstruction is indicated because of which of the following factors? A) Lateral defects do not need skeletal reconstructionB) Radiation is a contraindication for skeletal chest wall reconstructionC) Radiation renders the chest wall stiff, and thus large skeletal defects are well toleratedD) Reconstruction should be staged after margin confirmation on final pathologyE) Skeletal reconstruction should not be performed in oncologic resections

The correct response is Option C. The goals of skeletal chest wall reconstruction are restoration of chest wall stability and protection of intrathoracic structures. Large chest wall defects can result in flail segments with impairment of ventilatory mechanics. The lateral thoracic wall is most susceptible to such alterations since it the most mobile part of the chest wall. There is general consensus that skeletal chest wall reconstruction is indicated if four or more ribs are resected or if lateral defects are ? 5 cm. Total sternectomy defects disrupt the chest wall ring and increase dependence on abdominal breathing and may cause chronic chest wall pain; as such, they are considered by some to be indications for skeletal reconstruction. Posterior chest wall defects under the scapula (above the fourth rib) and anterior chest wall wounds under the pectoralis major do not require skeletal reconstruction since sufficient rigidity and protection is provided by these structures. Posterior defects around the area of the scapular tip may cause scapular entrapment and thus require skeletal reconstruction. Radiation fibrosis renders the chest wall stiff, and thus larger skeletal defects are well tolerated. Oncologic resections are not a contraindication for skeletal reconstruction. Skeletal reconstruction is most commonly performed with prosthetic materials, like synthetic meshes, biologic meshes and osteosynthesis materials. Radiation is not a contraindication for skeletal chest wall reconstruction. Reconstruction is performed at the same time as resection.

A 50-year-old woman with sarcoma of the right lateral chest wall has completed neoadjuvant radiation. Surgical resection is performed and results in a 6-cm diameter skeletal defect of the chest wall. Soft-tissue reconstruction only is planned. Soft-tissue only reconstruction is indicated because of which of the following factors? A) Lateral defects do not need skeletal reconstructionB) Radiation is a contraindication for skeletal chest wall reconstructionC) Radiation renders the chest wall stiff, and thus large skeletal defects are well toleratedD) Reconstruction should be staged after margin confirmation on final pathologyE) Skeletal reconstruction should not be performed in oncologic resections

The correct response is Option C. The goals of skeletal chest wall reconstruction are restoration of chest wall stability and protection of intrathoracic structures. Large chest wall defects can result in flail segments with impairment of ventilatory mechanics. The lateral thoracic wall is most susceptible to such alterations since it the most mobile part of the chest wall. There is general consensus that skeletal chest wall reconstruction is indicated if four or more ribs are resected or if lateral defects are ? 5 cm. Total sternectomy defects disrupt the chest wall ring and increase dependence on abdominal breathing and may cause chronic chest wall pain; as such, they are considered by some to be indications for skeletal reconstruction. Posterior chest wall defects under the scapula (above the fourth rib) and anterior chest wall wounds under the pectoralis major do not require skeletal reconstruction since sufficient rigidity and protection is provided by these structures. Posterior defects around the area of the scapular tip may cause scapular entrapment and thus require skeletal reconstruction. Radiation fibrosis renders the chest wall stiff, and thus larger skeletal defects are well tolerated. Oncologic resections are not a contraindication for skeletal reconstruction. Skeletal reconstruction is most commonly performed with prosthetic materials, like synthetic meshes, biologic meshes and osteosynthesis materials. Radiation is not a contraindication for skeletal chest wall reconstruction. Reconstruction is performed at the same time as resection. References

A 5-year-old male has a cerebrospinal fluid leak and a 3 x 3-cm area of wound dehiscence involving the posterior trunk following tethered cord repair. Which of the following is the most appropriate method to reconstruct the wound? A) Gluteal muscle flap and skin advancement flapB) Latissimus muscle turnover flap and skin advancement flapC) Local fascial flap and skin advancement flapD) Skin advancement flapE) Split-thickness skin graft

The correct response is Option C. The most appropriate method to reconstruct the wound is a local fascial flap and skin advancement flap. The major principle of tethered cord and myelomeningocele repair is to obtain a well-vascularized layer of soft tissue coverage between the dural and skin closures. The fascia overlying the paraspinous muscles can be turned over as flaps to cover the underlying dural repair. This vascularized soft tissue layer will minimize the risk of cerebrospinal fluid leak by reinforcing the dural repair. In addition, the fascial flaps will prevent contact with cutaneous bacteria and subsequent meningitis if either the dural repair or skin repair breaks down. A split-thickness skin graft over the dura would not adequately protect the spinal cord. Closing the skin directly over the dural repair using skin advancement flaps would place the child at risk for meningitis in the event of a cerebrospinal fluid leak or if wound breakdown occurred along the incision line of the widely undermined skin flaps. The use of a regional gluteal or latissimus muscle flap to cover the dural repair is unnecessary because local tissue (paraspinous muscle fascia) is available. Harvesting the gluteal or latissimus muscles also may cause significant donor site morbidity in a child already at risk for ambulatory problems from a neurological deficit.

A 5-year-old boy is evaluated for a cerebrospinal fluid leak and a 3 × 3-cm area of wound dehiscence involving the posterior trunk following tethered cord repair. Which of the following is the most appropriate method for reconstructing the wound? A) Gluteal muscle flap and skin advancement flapB) Latissimus muscle flap and skin advancement flapC) Local fascial flap and skin advancement flapD) Skin advancement flap onlyE) Split-thickness skin graft

The correct response is Option C. The most appropriate method to reconstruct the wound is a local fascial flap and skin advancement flap. The major principle of tethered cord and myelomeningocele repair is to obtain a well-vascularized layer of soft-tissue coverage between the dural and skin closures. The fascia overlying the paraspinous muscles can be turned over as flaps to cover the underlying dural repair. This vascularized soft-tissue layer will minimize the risk of cerebrospinal fluid leak by reinforcing the dural repair. In addition, the fascial flaps will prevent contact with cutaneous bacteria and subsequent meningitis if either the dural repair or skin repair breaks down. A split-thickness skin graft over the dura would not adequately protect the spinal cord. Closing the skin directly over the dural repair using skin advancement flaps would place the child at risk for meningitis in the event of a cerebrospinal fluid leak or if wound breakdown occurred along the incision line of the widely undermined skin flaps. The use of a regional gluteal or latissimus muscle flap to cover the dural repair is unnecessary because local tissue (paraspinous muscle fascia) is available. Harvesting the gluteal or latissimus muscles also may cause significant donor site morbidity in a child already at risk for ambulatory problems from a neurological deficit.

An otherwise healthy 14-year-old girl with Poland syndrome is brought to the office because of breast asymmetry and severe symptoms of right macromastia. She wears a size E cup brassiere and needs to fill the left side to attain symmetry in clothes. On examination, she has amazia and an absent sternal head of the pectoralis major muscle on the left side and macromastia of the right breast. Which of the following is the most appropriate reconstructive option for this patient? A) Left-sided breast reconstruction with a latissimus dorsi flap and right-sided reduction mammaplastyB) Left-sided breast reconstruction with a latissimus dorsi flap onlyC) Placement of a subcutaneous left-sided tissue expander and right-sided reduction mammaplastyD) No surgery until after puberty

The correct response is Option C. The most appropriate reconstructive option is to place a subcutaneous expander on the left side and perform a reduction of the right breast. This patient is still going through puberty and has a few years of growth left. However, her breast asymmetry is severe, symptomatic, and negatively impacting her life. Additionally, she has a good social support system with parents who are actively engaged in her well-being. Surgery that can potentially have permanent changes in a person's physical anatomy (e.g., scars), and potentially negative consequences (e.g., inability to breast-feed) must be performed with very detailed and complete informed consent, especially in minors. On the other hand, doing nothing could also lead to potentially negative social, emotional, psychological, and physical consequences. Therefore, if the patient and parents understand the risks and benefits, and are fully informed, surgical treatment is warranted in situations such as the one described. In this situation, simply placing an expander on the left side and not addressing the symptomatic right breast does not take care of all of her issues. A reduction of the right breast will not only help with her symptoms of breast hypertrophy, but will also enable a more symmetric reconstruction. Using a prosthesis to mimic an E cup breast is not ideal and will not succeed. Reducing the breast to about a C cup will allow the surgeon a realistic chance to reconstruct the left side to mimic the right breast. The expansion process on the left side would then continue until volume symmetry is attained compared with the right side. Through her growing years, the expansion process could continue. When she stabilizes in terms of her breast growth, the expander can then be substituted with a permanent prosthesis. A contralateral further reduction/lift can also be performed as needed. An ipsilateral latissimus muscle flap can also be performed simultaneously to recreate the anterior axillary fold or fill the infraclavicular space if grossly concave. However, the latissimus muscle flap in and of itself does not have the amount of tissue needed for larger breast reconstructions, as in this case, and needs a simultaneous expander placed under it for that reason.

A 65-year-old man is evaluated for a large mass in a previously irradiated area of the posterolateral chest. Preoperatively obtained ventilation-perfusion scans and pulmonary function testing are within normal limits. A full-thickness chest wall resection to include three ribs and at least 10 cm of each rib is planned. The soft-tissue defect is anticipated to be 20 cm in diameter. Which of the following factors in this patient is most likely to necessitate a rigid chest wall reconstruction? A) Posterolateral position of the defectB) Previous chest wall radiation therapyC) Total area of the chest wall defectD) Total number of ribs resected

The correct response is Option C. The most likely factor to indicate rigid chest wall reconstruction in this very large anticipated chest wall defect is the total area of the bony chest wall that is resected. Generally, reconstruction of the lateral bony chest wall seems necessary if four or more consecutive ribs are resected or if the diameter of the total defect is larger than 5 cm. Historically, protection of a flail segment of chest wall was based on the pendelluft principle, a phenomenon in which there is airflow back and forth between the lungs, resulting in underventilated segments of lung. This out-of-phase movement of the airway gas between the intact and flail-chest-side lungs was long believed to be the major contributor to respiratory dysfunction in patients with flail chest but has failed to be proven and appears to be a flawed hypothesis. In patients who have had radiation therapy, larger defects may be tolerated without rigid chest wall stabilization owing to fibrosis. As many as five ribs may be resected in patients who have undergone radiation therapy before reconstruction is considered because increased fibrosis produces chest wall stiffness. Anterior and posterior defects are typically better tolerated than lateral defects.

An otherwise healthy 14-year-old boy is evaluated because of bilateral breast enlargement over the past 3 months. He is worried that his friends will notice. He reports no illicit drug use and takes no medications. Physical examination shows normal hair distribution for the patient's age, no testicular masses, and firm discs of tissue under the areola of each breast. Which of the following is the most appropriate next step in treatment? A) Direct excision in the subareolar area, leaving a small button of tissue, and marginal breast liposuctionB) Liposuction of entire breast with use of cutting cannula under the areolaC) Reassessment in 9 monthsD) Testosterone replacementE) Wise pattern skin excision with nipple grafting

The correct response is Option C. The patient appears to have benign pubertal gynecomastia, and many of these cases resolve spontaneously. The recommended treatment is observation and reassessment. Early surgery in these circumstances is usually reserved for patients experiencing more extreme psychological impact. Testosterone replacement would not be useful since by examination the patient appears to have an otherwise normal physiology. The three surgical interventions are all potential treatments for gynecomastia, but surgery is usually not recommended within the first six months of onset of gynecomastia in a pubertal male. The skin excision and nipple grafting option would be most useful if the patient had large quantities of fat and skin. The two techniques employing liposuction both might be adequate surgical techniques to address the type of gynecomastia this patient demonstrates, should surgery eventually become recommended.

A 10-year-old boy is brought to the office because of the findings shown in the photograph. Which of the following is the most likely diagnosis? A) Amniotic band syndromeB) Axillary web syndromeC) Poland syndromeD) Popliteal pterygium syndromeE) Waardenburg syndrome

The correct response is Option C. The patient has Poland syndrome with an unusual axillary web. The defining clinical feature is underdevelopment or absence of the sternal head of the pectoralis major muscles (seen in the photograph), but the deformity can lead to other anomalies such as absence of the nipple, the areola, or portions of the anterior chest wall. Cardiac anomalies may also be present. The ipsilateral extremity and hand are often smaller or hypoplastic, and this finding can be subtle or pronounced. The fingers are typically shorter and smaller than the contralateral side, and there may be webbing between the fingers. In severe forms, the central fingers are mere vestiges. The other choices do not fit with this clinical photograph. Amniotic band syndrome usually leads to amputation or severe constriction of the hand or digits; this is not seen here. Waardenburg syndrome is a genetic condition that causes hearing loss and pigmentation changes to the hair, skin, and eyes. Popliteal pterygium syndrome is a condition associated with cleft lip/palate, webs of the popliteal space, and syndactyly of the toes or fingers. The axillary web seen here is not a described feature. Axillary web syndrome, or cording, is an acquired web that usually follows axillary node dissection in the treatment of breast cancer. This does not fit the clinical scenario.

A 22-year-old woman presents for consideration of aesthetic breast surgery to address asymmetry. Physical examination shows a unilateral hypoplastic breast with a constricted, elevated base and a herniated nipple-areola complex. Which of the following is the most likely diagnosis? A) AmastiaB) MicromastiaC) Poland syndromeD) Tuberous breastE) Virginal mammary hypertrophy

The correct response is Option D. A tuberous breast is classically defined as hypoplastic with a constricted and elevated base, insufficient inferior skin, and a herniated nipple-areola complex. Amastia would manifest without a nipple. Poland syndrome is classically described as missing the pectoralis muscle with variable breast and nipple effects. A constricted base and herniated areola are not usually associated with Poland syndrome. Hypertrophy would likely present with a broader base and increased volume. Micromastia would not manifest with a herniated areola.

A 45-year-old man undergoes abdominal reconstruction with a component separation technique. After release of the external oblique muscle bilaterally, which of the following best explains the preservation of flexion of the trunk? A ) Preservation of the nerves between the anterior rectus fascia and pyramidalis muscleB ) Preservation of the nerves between the external oblique and internal oblique musclesC ) Preservation of the nerves between the internal oblique and transversalis musclesD ) Preservation of the nerves between the transversus abdominis and the peritoneum

The correct response is Option C. The rectus muscle is the primary flexor of the pelvis. Its segmental nerve supply enters the rectus sheath laterally in a plane between the internal oblique and transversalis muscles. Separation of components is a versatile and powerful technique for autologous abdominal wall reconstruction. Along with the ability to close large midline ventral defects primarily, the rectus muscle remains innervated because only the aponeurosis of the external oblique is released. If correctly performed, the internal oblique layer will shield the underlying nerve supply to the rectus and result in a functional abdominal wall repair.

A 13-year-old boy is brought for evaluation by his mother because of development of his breasts during the past 3 months. The patient is at the 50th percentile for height and weight; BMI is 22 kg/m2. Physical examination shows no abnormalities except for moderately severe bilateral symmetric gynecomastia. Which of the following is the most appropriate next step in management?A ) LiposuctionB ) Liver function studiesC ) Observation and follow-up in 12 monthsD ) Surgical excisionE ) Testicular ultrasonography

The correct response is Option C. Three months is considered a relatively short duration for gynecomastia. Rohrich advocates waiting 12 months prior to considering surgery in the absence of other findings. Gynecomastia during puberty is common, and arbitrary use of laboratory tests in the absence of any clinical history or physical findings is not recommended. Careful history and physical examination are all that is required to identify pubertal gynecomastia. Reassurance and reassessment at 12 months are the most reasonable alternatives. If the patient still had gynecomastia that is not receding at 12 months, then surgery or liposuction would be a reasonable option. The use of other tests or biopsy is unnecessary and possibly dangerous.

A 32-year-old woman is scheduled to undergo subtotal reconstruction of the vagina using a pedicled deep inferior epigastric artery perforator flap after tumor ablation. Which of the following is the most likely advantage of using this type of flap coverage rather than the pudendal thigh (Singapore) flap? A) Better maintenance of sensory innervationB) Less vaginal bulkinessC) Less vaginal hair growthD) More optimal mucus secretionE) Single-staged procedure

The correct response is Option C. Vaginal reconstruction for congenital vaginal agenesis or after tumor resection remains a challenging reconstructive endeavor. The ideal technique provides a vaginal canal with adequate dimensions, texture, appearance, and sexual function without excessive donor site morbidity. The deep inferior epigastric artery perforator flap as a local regional flap has been described recently for vaginal reconstruction. It is a non-hair-bearing flap that is performed in a single stage but is insensate. Depending on the patient's body habitus, it is less bulky than the musculocutaneous flaps (gracilis, vertical rectus) used for vaginal reconstruction, but more bulky than the Singapore flap. Two perforators are included in the flap to ensure viability. The major disadvantage is a conspicuous donor site scar. The pudendal thigh flap, commonly referred to as the Singapore flap, is a local fasciocutaneous flap that has been widely used for neovaginal reconstruction. It is a thin, sensate flap based on the posterior labial arteries, which are a continuation of the perineal artery. Modifications of the flap have been described to enhance the viability and reach of the flap. It is a reliable, single-staged operation, but can result in endovaginal hair growth. This can be improved with preoperative electric or laser depilation of the vulvar portion of the flap. Neither flap secretes mucus. Both are single-staged operations.

A 41-year-old man is being evaluated prior to ventral hernia repair. History includes a traumatic abdominal injury with exposed bowel 4 years ago treated with negative pressure dressings and skin grafting. A separation of components technique will be used. The connection between which of the following layers will most likely remain intact during this procedure?A ) Bowel and fascial edgeB ) External oblique and internal obliqueC ) External oblique and subcutaneous layerD ) Internal oblique and transversus abdominisE ) Rectus muscle and posterior rectus sheath

The correct response is Option D. The principle of component separation is that the layers of the abdominal wall are able to be mobilized to a greater degree when they are separated from one another. In a ventral hernia repair, the bowel is freed from the fascial edges. Large skin flaps are developed that expose the abdominal fascia. The external oblique layer is separated from the internal oblique layer just lateral to the rectus sheath. Incising the posterior rectus sheath can add 1 to 2 cm of additional mobilization if necessary. The internal oblique muscle is usually not separated from the transversus abdominis muscle because it contains the intercostal nerves and blood vessels. This makes dissection difficult, bloody, and heightens the risk of denervating the rectus abdominis muscle.

A 24-year-old man is evaluated because of a 15-month history of painful idiopathic gynecomastia. Which of the following is the most appropriate treatment? A) AnastrozoleB) Radiation therapyC) SpironolactoneD) Surgical resectionE) Tamoxifen

The correct response is Option D. Adult gynecomastia occurs because of drugs, unresolved pubertal gynecomastia, and unknown causes (idiopathic gynecomastia). Tamoxifen and raloxifene, both selective estrogen receptor modulators (SERMs), can be used for the treatment of male gynecomastia. Indeed, the use of SERMs is recommended to prevent gynecomastia as a result of antiandrogen monotherapy for treatment of prostate cancer. They are most effective, however, when used to treat gynecomastia that has been present for less than 1 year. Anastrozole, an aromatase inhibitor, is recommended for the treatment of breast cancer and as a preventative measure in high-risk women. Surgical resection is the appropriate treatment for painful gynecomastia that has been present for greater than 1 year. Both open resection and liposuction are appropriate techniques. Radiation therapy is indicated to prevent gynecomastia as a result of prostate cancer treatment. Spironolactone is a cause of—not a treatment for—gynecomastia.

A 56-year-old man is evaluated for a ventral hernia after undergoing midline laparotomy for diverticulitis. BMI is 38 kg/m2. Physical examination shows midline fascial defect measuring 20 × 15 cm; there is no evidence of infection and skin coverage is stable. Repair with rectus advancement and polypropylene mesh is planned. Placement of mesh between which of the following planes is most likely to decrease this patient's risk of hernia recurrence? A) Anterior rectus sheath and rectus muscleB) Internal and external oblique musclesC) Medial edges of rectus muscleD) Rectus muscle and posterior sheathE) Skin and anterior rectus sheath

The correct response is Option D. Although recurrence rates are generally very high for large ventral hernias, placement of mesh in the retrorectal position appears to have the most decreased rate of recurrence compared with other methods. Placement above the plane of the rectus muscle requires division of vascular perforators that traverse the rectus muscle and perfuse the overlying skin flaps. These perforating branches of the epigastric circulation are most dense in the periumbilical zone. Preservation of the perforators has been shown to be beneficial in a number of case series reports. Placement of mesh between the oblique muscles in this case would not provide support for the midline hernia because these muscles are more lateral. Hernia recurrence rates are generally more increased in the presence of infection, with large defects when the rectus muscles cannot be advanced back together in the midline, in obese patients, and in patients with multiple medical comorbidities. Patients with several risk factors can expect recurrence rates in the range of 20 to 40%, whereas patients with few risk factors have recurrence about 5% of the time.

In bilateral component separation for abdominal wall reconstruction, which of the following points is most likely to be the area of greatest advancement? A) Arcuate lineB) Ligament of TreitzC) SuprapubicD) UmbilicusE) Xyphoid

The correct response is Option D. Component separation for abdominal wall reconstruction involves release of the fascia lateral to the rectus abdominus muscles, just lateral to the semilunar line, dissecting the external oblique off the internal oblique muscles. This creates innervated musculofascial flaps that can be advanced medially for closure of ventral hernias. If the posterior rectus sheath is also dissected free, further advancements can be gained. Per rectus muscle, approximately 4 cm can be gained at the epigastric and suprapubic areas; 10 cm can be advanced at the waist. Therefore, the most advancement can be gained at the umbilicus, which is in the area of the waist. The xyphoid and ligament of Treitz are in the epigastric area, while the arcuate line is in the suprapubic area.

A 12-week-old infant undergoes repair of a complete bilateral cleft lip. Which of the following is a principle of both bilateral and unilateral cleft lip repair? A) Centralizing the columellaB) Creating a rotation advancement flapC) Discarding the prolabial vermillionD) Establishing muscular continuityE) Preparing the projecting premaxilla

The correct response is Option D. Establishing orbicularis oris continuity is a goal in both unilateral and bilateral cleft lip repair. Centralizing the columella is performed only in unilateral cleft lip repair. The prolabium is only apparent in bilateral cleft lips. Preparing the projecting premaxilla is done only in bilateral cleft lip repairs. Creating a rotation advancement flap is performed only in unilateral cleft lip repair.

A 19-year-old man with no available medical history presents for initial evaluation of gynecomastia. Which of the following is most appropriate to include in this patient's workup? A) Chest x-ray studyB) Hand x-ray studyC) MammographyD) Testicular examination

The correct response is Option D. Gynecomastia can occur because of imbalances of estrogen and testosterone, and it can be associated with obesity, certain drugs, and Klinefelter syndrome. It can also be associated with testicular tumors. Therefore, a testicular examination should be performed. Mammography is not necessary for a routine workup if physical examination shows no abnormal masses. A chest x-ray study is not needed for an otherwise healthy 19-year-old. An x-ray study of the hand can evaluate closure of growth plates when determining if puberty has completed. This is unlikely to be necessary for someone who is 19 years old.

A 50-year-old man with a BMI of 36 kg/m2 comes to the office for consultation regarding gynecomastia. He takes no medications. Physical examination shows no other abnormalities. Which of the following is the most likely cause of this patient's gynecomastia? A) Decreased circulating estrogenB) Decreased estrogen receptorsC) Excessive androgen receptorsD) Excessive aromatization of androgen to estrogenE) Increased circulating androgen

The correct response is Option D. Gynecomastia is benign proliferation of breast tissue in men. It is present in 40 to 50% of men over 40 years of age. It can manifest in pubertal boys and in men of advanced years. The etiology can be variable and may be due to excess circulating estrogen, decreased circulating androgens, or a deficiency of androgen receptors. However, in middle-aged and older men, it is most commonly due to the excessive aromatization of androgens to estrogens. Initial evaluation requires a detailed history and physical examination to differentiate between fatty tissue, parenchymal enlargement, and a tumor. Mammography may be useful and biopsy may be indicated in some cases.

A 62-year-old man is evaluated for an 8-month history of fullness of the right breast associated with a subareolar mass. He reports no history of pain, nipple discharge, skin changes, or systemic symptoms. There is no family history of breast cancer. Examination of a specimen obtained on biopsy shows ductal epithelial hyperplasia with proliferation of stroma and fibroblasts. Which of the following is the most likely diagnosis? A) Ductal carcinomaB) Epidermal cystC) FibroadenomaD) GynecomastiaE) Lymphangioma

The correct response is Option D. Gynecomastia is defined as the benign proliferation of male glandular breast tissue. The most common symptom of gynecomastia is painless breast enlargement in adolescent and elderly men. Examination of a breast biopsy specimen may be necessary to exclude breast cancer if mammography and/or breast ultrasonography are suggestive of malignancy. Microscopic findings include ductal epithelial hyperplasia with proliferation of stroma and fibroblasts. Pseudogynecomastia is defined as breast enlargement secondary to fat deposition without glandular proliferation. Fibroadenoma are the most common benign tumor of the female breast. They are most often diagnosed in women between the ages of 20 and 35 years. Fibroadenomas are often ovoid or spherical, freely movable, and often well circumscribed. Microscopic findings include ductal tissue with proliferation of stroma surrounded by fibroblasts. Ductal carcinoma accounts for the majority of breast cancers. Ductal carcinoma is characterized microscopically by cords and nests of tumor cells with varying amounts of gland formation, and cytologic features that range from bland to highly malignant. Epidermal cysts are inclusion cysts lined by well-differentiated epidermis filled with keratin. Lymphangiomas of the breast are distended lymphatic channels interspersed with breast lobules.

A 63-year-old man with a BMI of 35 kg/m2 presents with an incisional hernia. The patient underwent a midline exploratory laparotomy for trauma one year ago. Primary fascial closure was achieved with a running polypropylene suture that was performed at the time of the initial operation. CT scan shows intact rectus muscles, and the hernia defect is measured to be 10 cm at the widest, which is in the supraumbilical region. Which of the following is the most effective treatment to prevent hernia recurrence following repair? A) Component separation with bridging mesh repairB) Component separation with overlay mesh repairC) Component separation with primary fascial closureD) Component separation with retrorectus mesh repairE) Primary fascial closure

The correct response is Option D. Hernia repair is associated with a high rate of recurrence, approaching 20% in many studies. Recurrence rates are lowest when primary fascial closure of the abdominal wall is reinforced with mesh placement as an underlay. Primary fascial closure alone or with component separation results in a higher recurrence rate than primary fascial closure with mesh reinforcement. In this example, it is unlikely that primary fascial closure would be possible, given a 10-cm hernia defect. With regard to mesh placement, there are multiple planes at which the mesh can be placed. Using a bridging repair, the mesh is used to bridge across a fascial defect and is associated with the highest rates of recurrence. In a retrorectus repair, the mesh is placed deep to the rectus (Rives-Stoppa technique) or below the transversus abdominis (transversus abdominis release technique). This is performed underneath a primary fascial closure. Conversely, in an overlay repair, the mesh is secured superficial to the abdominal wall repair. Retrorectus placement of a mesh is associated with a significantly lower recurrence rate than placement of the mesh in another position.

A 65-year-old man is prescribed leuprolide acetate for prostate cancer. Which of the following is the most likely effect the drug will have on this patient's breasts? A) Darkening of the nipple-areola complexB) Decrease in sizeC) GalactorrheaD) MastodyniaE) Petechiae

The correct response is Option D. Leuprolide acetate (Lupron) is used in the treatment of certain cancers, including prostate, endometriosis, and precocious puberty. Because it is a synthetic gonadotropin-releasing hormone (GnRH), it acts as an agonist of pituitary GnRH receptors. Its ultimate effect is a lowering of estradiol and testosterone levels through downregulation of luteinizing hormone and follicle-stimulating hormone secretion. Gynecomastia and breast tenderness are known side effects of leuprolide treatment for prostate cancer. Selective estrogen receptor modulators (SERMs), such as tamoxifen, are recommended for the prevention of gynecomastia as a result of antiandrogen monotherapy. Radiation as well has been shown to decrease the breast side effects of leuprolide acetate. Thus, an increase in size, rather than a decrease in size is expected. Darkening of the nipple areolar complex is often associated with pregnancy, but not leuprolide acetate. Galactorrhea is associated with drugs—commonly drugs that contain dopamine depleting agents, such as methyldopa, that cause receptor bock (such as Reglan), and that inhibit release, such as codeine and morphine. Histamine receptor blockade, from drugs such as cimetidine, can also cause galactorrhea. Pituitary tumors are also a cause of galactorrhea. A thorough assessment of all medications and medical history is necessary for evaluation of a patient with gynecomastia and mastodynia.

A 6-year-old girl with bilateral cleft lip and palate presents to the office with velopharyngeal dysfunction. Secondary speech surgery is planned. Which of the following procedures carries the highest risk for developing postoperative obstructive sleep apnea? A) Conversion Furlow palatoplastyB) Palatal lengthening with bilateral buccal flapsC) Repeat palate repair with intravelar veloplastyD) Sphincter pharyngoplastyE) Velopharyngeal augmentation with structural fat grafting

The correct response is Option D. Multiple techniques have been described for management of velopharyngeal dysfunction (VPD) following primary palatoplasty in patients with cleft palate and cleft lip/palate. These include pharyngoplasty techniques, such as posterior pharyngeal flaps and sphincter pharyngoplasties. These procedures add tissue to the velopharyngeal port and partially occlude airflow. These procedures are the most effective in correcting VPD but carry with them the highest risk for postoperative obstructive sleep apnea (OSA). The incidence of postobstructive OSA following sphincter pharyngoplasty varies between 50 to 69% of patients. Other procedures described include palatoplasty techniques such as intravelar veloplasties, conversion Furlow palatoplasties, and palatal lengthening procedures. These procedures have been shown to correct VPD in 40 to 80% of cases and have varying risks for postoperative OSA, varying from 10 to 54%. Finally, augmentation of the velopharyngeal port with structural fat grafting is successful in correcting mild cases of VPD, but it was recently shown to carry a small (11%) risk for postoperative OSA. Treatment of VPD is nuanced, and all techniques should be considered. Patient factors such as the history of prior palate surgery, physical examination, dynamic assessment of velopharyngeal port function, and patient comorbidities must be considered to select the appropriate procedure in every case.

A 28-year-old man presents for consultation about the excess tissue of his chest. Medical history includes no weight change and no comorbidities. BMI is 26 kg/m2. Physical examination shows the nipple position is preserved in the craniocaudal dimension and firm soft tissue underlying the central breast bilaterally. Which of the following is the most appropriate surgical approach? A) Horizontal scar breast reduction with nipple-areola graftingB) Inverted-T pattern breast reduction with inferior pedicleC) Liposuction aloneD) Periareolar access for open central glandular excision with liposuctionE) Skin-sparing mastectomy with nipple-areola tattoo

The correct response is Option D. Scar minimization is a priority when possible in patients with gynecomastia. The patient's acceptable BMI, absence of weight change, and physical description imply minimal skin excess, making him a candidate for limited scar techniques. Direct excision of the central gland through periareolar incision with liposuction for marginal contouring would address the patient's condition without excess scarring. Liposuction alone is unlikely to adequately address the firm central glandular tissue under the nipple-areola complex. The other techniques such as inverted-T pattern breast reduction with inferior pedicle, horizontal scar breast reduction with nipple-areola grafting, and skin-sparing mastectomy with nipple-areola tattoo would all unnecessarily increase scarring.

A 58-year-old man comes to the office with recurrent rectal cancer four years after undergoing low anterior resection with adjuvant radiotherapy. Open abdominoperineal resection is planned. Which of the following closure methods will most reliably reduce the incidence of perineal wound complications? A) Gracilis flap closureB) Negative pressure wound therapyC) Primary closureD) Rectus abdominis flap closureE) Split-thickness skin grafting

The correct response is Option D. Several retrospective studies and one randomized trial have shown that when compared to primary closure, the rectus abdominis myocutaneous flap reduces wound healing complications after abdominoperineal resection (APR). Gluteal and gracilis flaps have been used for reconstruction after APR; however, the data supporting their use is not as robust. Split-thickness skin grafting and negative pressure wound therapy are inappropriate for reconstruction of the APR due to the size of the wounds and the risk of evisceration.

The plastic surgeon is called to the neonatal unit to evaluate a newborn with a myelomeningocele. There is an intact 4 × 4-cm sac at the lumbosacral area with minimal fluid oozing from the area. Early operative repair is indicated for which of the following reasons? A) To decrease the need for cerebrospinal fluid shunt placementB) To decrease the need for folic acid supplementationC) To improve motor function returnD) To prevent bacterial meningitisE) To prevent cerebrospinal fluid leakage

The correct response is Option D. The major indication to repair a myelomeningocele defect in the early postnatal period is to prevent infection and bacterial meningitis. Great care is taken to keep the sac sterile and hydrated. Further cardiac, orthopedic, and urologic workup is often necessary, as well as evaluation for hydrocephalus. Although improvement in the return of motor function with early repair has not been shown, neurogenic bladder prognosis is improved. Leakage of cerebrospinal fluid is commonly observed regardless of timing of closure. Although folic acid supplementation has been shown to decrease the neural tube defects and myelomeningocele formation, postnatal supplementation has not been shown to be effective. Approaches to surgical therapy for the treatment of these defects have seen a shift toward prenatal, fetal reconstructive surgery. A recent study by Adzick et al. in the New England Journal of Medicine examined the use of prenatal surgery versus postnatal surgery. In this randomized trial, the authors found a decreased need for cerebrospinal fluid shunt placement and improved motor function outcomes.

An otherwise healthy 25-year-old man comes to the office because of a 10-year history of bilateral gynecomastia. Physical examination shows breast enlargement with skin redundancy and palpable glandular and fatty tissue. Which of the following is the most appropriate next step in management? A) Determination of 17-ketosteroid level in urineB) MammographyC) Referral to an endocrinologistD) Surgical excisionE) Testicular ultrasound

The correct response is Option D. The most appropriate next step for the patient described, who is young with a long history of bilateral gynecomastia since puberty, is surgical excision. The onset of gynecomastia correlates with transient elevations of plasma estradiols prior to the completion of puberty so that the androgen-to-estrogen ratio is altered. Suction lipectomy can also be used as surgical treatment for gynecomastia. Most cases of gynecomastia present at puberty, with an incidence as high as 65% in boys 14 to 15 years of age. The condition disappears during the late teens, with only 7.7% remaining at age 17 years. The incidence rises again with progressive age. The condition is often a normal finding, even though it may be associated with a more serious disease in occasional cases. In certain cases, systemic causes — such as liver disease, lung carcinoma, testicular carcinoma, adrenal tumors, thyroid disease, testosterone imbalance, and Klinefelter syndrome — or drugs like marijuana, should be considered. For instance, a prepubescent boy presenting with gynecomastia would cause concern. Another example would be if an adult man presented with a 6-month history of unilateral gynecomastia. A work-up including liver function test, urine studies, testicular examination, endocrinology evaluation, and possibly mammography should be done if there is a possibility of cancer (ie, patients with Klinefelter syndrome).

A 1-day-old female newborn is evaluated because of a 4 × 4-cm defect after undergoing dural repair of a lumbar myelomeningocele. Which of the following is the most appropriate next step in management? A) Negative pressure wound therapyB) Split-thickness skin graftingC) Coverage with a skin advancement flapD) Coverage with a local musculofascial flapE) Coverage with a free flap

The correct response is Option D. The most appropriate next step in management is coverage with a local musculofascial flap. The major principle of myelomeningocele repair is to obtain a well-vascularized layer of soft-tissue coverage between the dural repair and skin repairs. The fascia overlying the paraspinous muscles can be turned over as flaps followed by paraspinous muscle advancement flaps to cover the underlying dural repair. This vascularized soft-tissue layer will minimize the risk of contact of cerebrospinal fluid with cutaneous bacteria and subsequent meningitis if either the dural repair or skin repair breaks down. Vacuum-assisted wound therapy over a dural repair would increase the risk of cerebrospinal fluid leak, dural injury, and breakdown of the repair. A split-thickness skin graft over the dura would not adequately protect the spinal cord. Closing the skin directly over the dural repair would place the patient at risk for meningitis in the event of a cerebrospinal fluid leak, or if wound breakdown occurred along the incision line of the widely undermined skin flaps. The use of a free flap to cover the dural repair is unnecessary because local tissue (paraspinous muscles and fascia) is available. Harvesting the gluteal muscle(s) may cause significant donor site morbidity in a child already at risk for ambulatory problems from a neurologic deficit.

A 32-year-old woman comes to the office for consultation regarding gender reassignment surgery. She is in a stable relationship with a supportive partner and has the support of her family. She is interested in mastectomy, hysterectomy, salpingo-oophorectomy, and phalloplasty. Referral to which of the following is the most appropriate next step in management? A) Breast surgeon for mastectomyB) Endocrinologist for hormonal manipulationC) Gynecologist for hysterectomy and salpingo-oophorectomyD) Psychiatrist for evaluationE) Urologist to assist with phalloplasty

The correct response is Option D. The most appropriate next step in management is referral to a psychiatrist for evaluation. Gender identity disorder, formerly known as transsexualism, should ideally be managed by a multidisciplinary team. Ideal management includes psychotherapeutic, hormonal, and surgical treatments. Initial care involves a diagnostic phase in two stages. The goal of the first stage is to establish the diagnosis according to well-defined criteria and to rule out other psychiatric disorders. The second stage of the diagnostic phase is a ?real-life test,? in which the patient assumes the role of the opposite sex. Ongoing supportive psychotherapy is often required. Some centers begin hormonal therapy during the ?real-life test? to assist the transition and to make the test more realistic. Most centers defer hormonal therapy until 1 year following initial contact with the patient; surgery is deferred for 2 years following contact. There are no well-established protocols to guide the surgical sequence.

A 59-year-old man with hypertension, peripheral vascular disease, and coronary artery disease has sternal osteomyelitis after coronary artery bypass grafting with saphenous vein and left internal mammary artery grafts. Reconstruction with bilateral pectoralis advancement flaps is performed, but the flaps do not survive. Debridement is performed, and a defect remains. A photograph is shown. Which of the following is the most appropriate reconstruction? A ) Adjacent tissue transfer and coverage with bilateral skin advancement flapsB ) Coverage with left rectus abdominis turnover flap and skin graftingC ) Coverage with left superior epigastric artery perforator fasciocutaneous flapD ) Coverage with omental flap and skin graftingE ) Coverage with right latissimus myocutaneous flap

The correct response is Option D. The most appropriate option in the patient described is an omental flap with skin grafting. The defect encompasses the entirety of the sternum, from the sternal notch to the xiphoid. The first-line muscle flaps for this defect would usually be bilateral pectoralis muscle flaps (that do not include a left pectoralis turnover flap option due to the lack of a left internal mammary artery (IMA) used for the coronary graft). However, this option has already been used. An omental flap has an axial blood supply that is not compromised (the gastroepiploic) and the ability to completely span the defect and obliterate the dead space; the skin graft provides the skin coverage needed to complete the reconstruction. Hence, it is the optimal choice in this patient. An adjacent tissue transfer of random skin flaps bilaterally would not obliterate the dead space and would not provide robust axial blood supply to the necessary area, with the tenuous coverage existing along the sternal midline, exactly where the most vital blood supply would be necessary. A left rectus abdominis turnover flap would not be an appropriate option for two reasons: 1) the left IMA has been harvested, thereby compromising the superior epigastric vessel on which this flap would be based; and 2) it would not reach the upper portion of the defect. A left superior epigastric artery perforator fasciocutaneous flap would not be an appropriate option for two reasons: 1) the pedicle is compromised from the left IMA harvest; and 2) muscle flaps have been proven to be optimal for sternal defects more than fasciocutaneous flaps. A right latissimus muscle flap may be a viable choice to get full coverage of the defect with dead space obliteration. However, the option indicates this is a myocutaneous flap, which would not allow for sufficient skin to be harvested to reconstruct the size of this defect.

An otherwise healthy 55-year-old woman is diagnosed with squamous cell carcinoma of the vagina. Subtotal vaginectomy, vulvectomy, and inguinal lymph node dissection are performed. Bilateral sensate posterior thigh flaps are used for vaginal reconstruction. Which of the following nerves must be included in this procedure? A ) IliohypogastricB ) IlioinguinalC ) Inferior glutealD ) Posterior femoral cutaneousE ) Pudendal

The correct response is Option D. The nerve that gives sensation to the posterior thigh flap is the posterior femoral cutaneous nerve (S1 to S3). This fasciocutaneous flap is based on the descending branch of the inferior gluteal artery and provides an abundant and reliable amount of available soft tissue for transfer from the posterior aspect of the thigh. Because of the proximal vascular pedicle, this flap can easily reach high within the pelvis and can be rotated to provide closure to both anterior and posterior vaginal defects. In addition, the flap can also be harvested with relative technical ease in a short period of time, and the donor site can typically be closed without the need of skin grafts. The skin is innervated by the posterior femoral cutaneous nerve (S1 to S3), which allows this flap to be used as a sensate flap when indicated. There are several other advantages to the posterior thigh flap that make it ideal for complex perineal reconstruction. The inferior gluteal artery is a terminal branch of the internal iliac artery. Therefore, the

An otherwise healthy 25-year-old man comes to the office with concerns about recent enlargement of his breasts. BMI is 28 kg/m2 and has not changed during the past year. He does not take any prescription or illicit drugs. He has no nipple discharge. Examination of which of the following is most appropriate? A) Cervical lymph nodesB) Cranial nervesC) Peripheral pulsesD) ProstateE) Scrotum and testes

The correct response is Option E. The patient described is likely to have gynecomastia, which can occur in up to 30% of men. In the younger, postpubertal patients, testicular cancer may be a cause. Examination of the scrotum and testes to rule out any testicular masses has to be included in the physical examination and documented. Measurement of beta-human chorionic gonadotropin concentration can also be effective. The other examinations listed do not directly contribute to the assessment of gynecomastia.

A 52-year-old woman with cancer of the right breast undergoes mastectomy and axillary node dissection, complicated by mastectomy flap necrosis requiring skin grafting. She completes adjuvant chemoradiation. One year later, she comes to the office with a fungating mass growing through the skin graft. Imaging demonstrates involvement of the fourth and fifth ribs with an anticipated skeletal defect of 4 × 4 cm. A photograph is shown. Which of the following is the most appropriate treatment for the skeletal reconstruction? A) High-density porous polyethyleneB) Methyl methacrylate with meshC) 2.4-mm Titanium plateD) No skeletal reconstructionE) Vascularized rib

The correct response is Option D. The principles of management of this recurrent right breast cancer include radical resection of all involved tissues (including ribs) and reconstruction with well vascularized flaps. In this case, a right latissimus muscle flap and skin graft was used for reconstruction. No alloplastic material was placed or skeletal thoracic cage reconstruction performed. This is common in these types of patients, because excessive fibrosis caused by the radiation to the chest wall prevents loss of respiratory efficiency through paradoxical motion which otherwise occurs in patients who have more than four ribs involved or a defect larger than 5 cm.

A 56-year-old man is evaluated because of gynecomastia. Physical examination shows mild, diffuse breast enlargement with no visible inframammary fold or ptosis. Which of the following is the most appropriate surgical correction of this patient's condition? A) En bloc resection of skin and breast tissue with free nipple graftingB) Open excision of breast tissue with mastopexyC) Subcutaneous mastectomy with nipple preservationD) Suction-assisted lipectomyE) Superior periareolar excision with skin excision

The correct response is Option D. The treatment of gynecomastia is based on the degree of breast enlargement and the extent of ptosis that is noted on examination. Grade 1 gynecomastia is minimal breast hypertrophy without ptosis. Grade II gynecomastia is moderate hypertrophy without ptosis. Grade III gynecomastia is severe hypertrophy with moderate ptosis. Grade IV gynecomastia is severe hypertrophy with severe ptosis. The treatment of mild to moderate gynecomastia without ptosis is suction-assisted lipectomy. Direct periareolar excision with skin excision and subcutaneous mastectomy are not indicated for gynecomastia without ptosis. Mastopexy and free nipple grafting techniques are indicated for gynecomastia with severe ptosis.

Which of the following is the most appropriate term for the amount of air that is inspired and expired with a single breath during normal resting respiration? A) Dead-space volumeB) Functional capacityC) Residual volumeD) Tidal volumeE) Vital capacity

The correct response is Option D. Tidal volume (VT) is the volume of air that is moved into or out of the lungs during quiet breathing. Tidal volume can be measured directly through spirometry or estimated based on a patient's ideal body mass. It is a key parameter in mechanical ventilation to allow adequate ventilation without causing barotrauma to the lungs. Vital capacity (VC) is the volume of air expired after deepest inspiration. Functional capacity is a physiologic description of an individual's ability to complete activities of daily living. It can be estimated through exercise treadmill testing and reported in metabolic equivalents (METs). Residual volume (RV) is the volume of air remaining in the lungs after maximal exhalation. Dead-space volume is the volume of air inhaled that does not take part in gas exchange. This volume can include both gas that remains in conducting airways (e.g., trachea, bronchi) during respiration and gas that reaches nonfunctional alveoli (e.g., nonperfused lung parenchyma following pulmonary embolism).

A 66-year-old man with a history of adenocarcinoma of the lung undergoes a left pneumonectomy using a posterolateral thoracotomy incision. Postoperatively, he receives radiation therapy. The patient subsequently develops a nonhealing ulcer of the chest wall measuring 5 × 6 cm in the region of the nipple-areola complex. Which of the following is the most appropriate option for reconstruction? A) Negative pressure wound therapyB) Pedicled intercostal muscle flap and a split-thickness skin graftC) Pedicled latissimus dorsi musculocutaneous flapD) Pedicled vertical rectus abdominis musculocutaneous flapE) Split-thickness skin graft only

The correct response is Option D. Tissue injury from radiation results in irreversible damage that limits the ability of wounds to heal with skin grafts or by secondary intention, such as using negative pressure wound therapy. Tissues within the field of radiation are also generally affected, which would limit use of an intercostal muscle flap. Although a latissimus dorsi musculocutaneous flap would provide an adequate amount of tissue from outside of the field of radiation, it would be unreliable in the setting of prior posterolateral thoracotomy, unless there is documentation that the latissimus was spared. A vertical rectus abdominis musculocutaneous flap would provide sufficient tissue from outside of the field of radiation and would not have been affected by the patient's prior surgery.

An 83-year-old woman comes to the office for repair of a deep sternal wound infection after undergoing open single-vessel coronary artery bypass grafting using the left inferior mammary artery. Medical history includes cancer of the left breast for which she underwent wide excision with oncoplastic reconstruction followed by adjuvant radiation therapy to the breast and axilla. She does not smoke cigarettes. The patient's infection resolves with aggressive debridement and culture-guided antibiotics. A photograph is shown. Which of the following is the most appropriate reconstructive option? A) Left pectoralis turnover flap coverage with skin graft resurfacingB) Left vertical rectus abdominis musculocutaneous flap coverageC) Right latissimus dorsi musculocutaneous flap coverageD) Right pectoralis advancement flap with skin readvancement closureE) Right vertical rectus abdominis musculocutaneous flap coverage

The correct response is Option E. Deep sternal wound infections represent life-threatening infections whose most frequent etiology in contemporary cases is surgical site infection following open heart surgery, with a reported incidence of 0.2 to 3% of patients undergoing median sternotomy. Risk factors in adults include diabetes mellitus, obesity, peripheral arterial disease, tobacco use, reoperation, and other surgical complications such as prolonged operative time and postoperative bleeding. Published mortality rates range from 8.1 to 14.8%. Once a diagnosis is made, initial treatment in cases of deep infection typically involve a combination of antimicrobial therapy and staged surgical debridement followed by delayed closure. Sternotomy wounds can be complex problems because of their proximity to heart and lungs and exposure of these vital organs following debridement. Principles of reconstruction once a healthy wound bed has been obtained include durable coverage of vital structures with obliteration of dead space with a view to retaining or restoring optimal form and function. In the case scenario described, a right superiorly based rectus abdominis myocutaneous flap provides the most reliable, well vascularized and non-irradiated option for regional soft tissue reconstruction of the composite chest wall defect (shown). A left vertical rectus abdominis myocutaneous flap is suboptimal in the setting of a relatively compromised superior epigastric vascular pedicle in the setting of known sacrifice of its parent left internal mammary artery (IMA). Latissimus and pectoralis flaps in any form are less reliable in their ability to fully cover inferior third sternectomy defects. While a turnover pectoralis flap can reach the inferior sternum, a left pectoralis turnover flap is furthermore less reliable in the setting of known sacrifice of its inferior segmental internal mammary perforators in the setting of known IMA sacrifice. Skin re-advancement in the setting of prior radiation and distorted perfusion from prior reduction mammoplasty also invites further unnecessary risk.

A 22-year-old man comes to the office reporting breast enlargement that began 7 years ago. Physical examination shows bilateral breast enlargement. Testicular examination is normal. BMI is 27 kg/m2. Which of the following evaluations is best for distinguishing gynecomastia from pseudogynecomastia in this patient? A) History of medication useB) Laboratory testingC) MammographyD) MRIE) Physical examination

The correct response is Option E. Gynecomastia is benign enlargement of the glandular breast tissue in a man. It is often related to factors that either results in an increase in estrogen production, or a decrease in androgen production. This can be due to the transient imbalances during puberty, or secondary to medication use, drug use (such as marijuana), testicular pathology, thyroid disease, liver disease, breast cancer, etc. This is in contrast to pseudogynecomastia, which is enlargement of the breast due to fat deposition in the absence of glandular hypertrophy. Pseudogynecomastia is typically seen in the patient who is overweight or obese. Pseudogynecomastia is distinguished from gynecomastia by physical examination. In true gynecomastia, one will palpate enlarged firm glandular breast tissue, as opposed to in pseudogynecomastia, where palpation will reveal a soft fatty breast throughout with no enlargement of the subareolar tissue. History may suggest a pathologic etiology that will be confirmed on laboratory testing. Medication and drug use questioning will show if the gynecomastia is pharmacologic in origin. Mammography and MRI will aid in determining if there is a malignant etiology causing the breast tissue enlargement. However, it is not standard for gynecomastia evaluation.

A 56-year-old woman who has type 2 diabetes mellitus is scheduled to undergo removal of prosthetic mesh that became infected after ventral hernia repair. Weight is 311 lb (141 kg); BMI is 53 kg/m2. Reconstruction with component separation technique and onlay grafting of reinforcing mesh is planned. Compared with polytetrafluoroethylene, acellular dermal matrix is preferable for this patient because of its ability to undergo which of the following processes?A ) ContractionB ) EncapsulationC ) ImbibitionD ) InosculationE ) Revascularization

The correct response is Option E. Implant infection is a common clinical complication of abdominal hernia repair. Use of biologic mesh in contaminated cases has gained widespread acceptance. The ability of acellular dermal matrix (ADM) to revascularize is most likely the key to its successful application in contaminated scenarios. Revascularization subsequently helps clear bacteria. Contraction is a normal part of wound healing mediated by myofibroblasts, which are not present in polytetrafluoroethylene (PTFE) or ADM. Encapsulation is a foreign body reaction, whereby the organism forms a sterile scar around an alloplastic material. Imbibition is the process by which autogenous grafts perform initial metabolic exchange through diffusion. Inosculation is the first stage of neoangiogenesis, whereby immature vasculature between graft and recipient bed begin to organize in preparation for ingrowth. Neither process occurs in PTFE or ADM, as there is no existing vasculature in either product.

A 57-year-old woman undergoes resection of a squamous cell carcinoma of the vagina, resulting in a defect of the posterior two thirds of the vaginal vault from the introitus to the dome. A small portion of the rectum is involved in the resection and primarily repaired. The patient has a history of pelvic radiation therapy and tobacco use. Which of the following is the most appropriate approach for closure of this defect? A) Bilateral gracilis muscle flapsB) Deep inferior epigastric artery perforator flapC) Primary repair of the vaginectomy defectD) Split-thickness skin grafting with obturatorE) Vertical rectus abdominis musculocutaneous flap

The correct response is Option E. In the clinical scenario described, the vertical rectus abdominis musculocutaneous (VRAM) flap is the most appropriate choice. This approach can provide enough tissue to resurface the vaginal vault and fill dead space. For posterior defects of the vaginal vault, abdominal-based flaps are usually preferable. They can provide a large amount of vascularized tissue that rotates easily into the defect. Bilateral gracilis muscle flaps alone would offer less soft tissue, and mucosalization in a radiated field is unpredictable. A deep inferior epigastric artery perforator flap is far more complicated than a VRAM flap, and the donor site is problematic in patients with a history of smoking. Primary closure of a large vaginal defect would likely cause significant stenosis and poor healing because of previous radiation. A skin graft with obturator is not likely to be successful given her radiation and tobacco history and violation of the rectum.

A 27-year-old woman is scheduled to undergo female-to-male transgender surgery. A fasciocutaneous radial forearm flap is designed to construct the phallus. This procedure includes neurorrhaphy of the lateral antebrachial cutaneous nerve to a terminal branch of which of the following? A) Genitofemoral nerveB) Iliohypogastric nerveC) Ilioinguinal nerveD) Posterior femoral cutaneous nerveE) Pudendal nerve

The correct response is Option E. In women, the pudendal nerve terminates in the dorsal nerve of the clitoris, which is responsible for most erogenous sensation. Neurorrhaphy to this nerve has been shown to result in superior sensory outcomes in phalloplasty. The dorsal nerve of the clitoris is analogous to the dorsal nerve of the penis, which is used as a target for neurorrhaphy during penile reconstruction for amputation or congenital microphallus. Other branches of the pudendal nerve include the inferior rectal and perineal nerves. The ilioinguinal nerve supplies sensation to the skin of the upper medial thigh and to the skin of the mons pubis and labia majora. The genitofemoral nerve also provides sensation to the labia majora and skin of the femoral triangle. The posterior femoral cutaneous nerve innervates the skin of the posterior surface of the thigh as well as part of the perineum. The iliohypogastric nerve innervates the skin above the pubis and in the gluteal region.

A 43-year-old woman brings her 10-day-old daughter for evaluation because she is concerned about the appearance of her daughter's nipples. The newborn was born at full-term after an uncomplicated pregnancy. Physical examination shows that the newborn has inverted nipples bilaterally. Which of the following is the most appropriate next step in management? A) Continuous elastic outside distraction starting at day of life 14B) Suction lengthening of the nipple starting at day of life 14C) Suction lengthening of the nipple starting immediatelyD) Transection of the fibrotic bands at 6 months of ageE) Observation

The correct response is Option E. It is very common for nipples to be inverted at birth and concerned parents often discuss the condition with pediatricians and practitioners. The mammary ridge begins to develop in the fourth week of development, and produces the primary and secondary buds. The mammary pit is a small depression present at birth into which the lactiferous ducts open. After birth, proliferating mesoderm during the first several weeks of life will cause the inverted nipple to protrude. In some cases, the inverted nipple persists. Although there may not be a functional consequence, it can lead to difficulty with lactation. Han and Hong described a grading system for inverted nipple: grade I, in which the nipple can be made to protrude manually and can maintain position without traction; grade II, in which the nipple fails to maintain projection after manipulation; and grade III, in which the nipple cannot be pulled out manually. Treatment is based on the grade. In some cases, breast-feeding or use of a breast pump is sufficient to correct the variation. Surgical intervention can be approached in several ways. Constricting the base of the inverted nipple can create projection, severing the ducts can cause projection, and myotomy can cause projection. Any division of the ducts will compromise a patient's ability to breast-feed.

A 2-week-old male newborn with a wide unilateral cleft lip and palate presents for evaluation and surgical treatment planning. To align the alveolar segments, decrease the cleft width, and improve the nasal contour, nasoalveolar molding versus lip adhesion is discussed with his parents. The primary advantage of lip adhesion over nasoalveolar molding is which of the following? A) Decreased costB) Decreased scar tissueC) Fewer treatment complicationsD) Improved clinical outcomeE) Increased compliance

The correct response is Option E. Lip adhesion is the surgical version of nasoalveolar molding (NAM) in terms of accomplishing decreased cleft width and alveolar alignment; however, it does not do much for the nasal contour. The primary advantage of lip adhesion surgery is that it minimizes the burden on the family. It costs more, potentially can create more scar, and has a higher complication rate in the form of either scar or wound dehiscence. There is no proven clinical outcome advantage, and some would argue that because NAM also addresses the nose, the outcome is better, but that will take additional studies to validate.

A 45-year-old woman with rectal cancer is scheduled to undergo abdominoperineal resection and posterior vaginectomy following neoadjuvant chemotherapy and radiation therapy. Which of the following is the most appropriate method of reconstruction of the posterior vaginal defect? A) Free anterolateral thigh fasciocutaneous flapB) Primary repair of the perineal and vaginal defectsC) Pudendal thigh (Singapore) flapD) Skin graftingE) Vertical rectus abdominis myocutaneous flap

The correct response is Option E. Options for reconstruction of this irradiated, complex, three-dimensional defect include various flaps, but the best choice is the vertical rectus abdominis myocutaneous (VRAM) flap, due to its ease of harvest, minimal donor-site morbidity, large skin paddle (which can resurface both the perineum and the vagina), reliable blood supply, and favorable arc of rotation. Although the pedicled ALT flap has recently been described for vaginoperineal defects, the VRAM flap remains the gold standard. Primary repair, even if technically possible, would be associated with unacceptable risk of wound-healing problems.

In addition to the typical chest wall deformities, which of the following is the most common associated clinical presentation of patients suffering from Poland syndrome? A) DextrocardiaB) Lung herniaC) Renal malformationD) Thyroid malignanciesE) Upper extremity anomalies

The correct response is Option E. Poland syndrome is the rare congenital condition describing the absence of the breast or nipple, hypoplasia of subcutaneous tissue, absence of the costosternal portion of the pectoralis major muscle, absence of the pectoralis minor muscle, and absence of costal cartilages or ribs 2, 3, and 4 or 3, 4, and 5, occasionally even rib 6. Its aesthetic and functional impairment varies between individuals and highly depends on the severity of the disease. Boys are three times as likely to be affected as girls. Incidence ranges between 1:100,000 to 1:7,000 cases per live births. Various concomitant anomalies have been published in association with Poland syndrome that typically necessitate a multi-disciplinary approach towards the diagnosis and treatment of this congenital illness. Upper extremity anomalies, renal malformation, lung hernia, and dextrocardia have all been described in literature as related to Poland syndrome. Anomalies of the upper extremity are the most common concomitant malformation besides the chest wall defects. References

A 17-year-old girl comes to the office for consultation regarding augmentation mammaplasty. Physical examination shows right-sided hypoplasia of the chest wall and breast. Absence of the pectoralis major muscle is noted. This patient is most likely to have which of the following hand anomalies? A) MicromeliaB) OligodactylyC) PhocomeliaD) PolydactylyE) Syndactyly

The correct response is Option E. Poland syndrome was described by Sir Alfred Poland in 1841 after a cadaver dissection while he was a medical student. His initial report described the absence of pectoralis major and minor musculature and upper extremity anomalies. Absent in his account were other components of the syndrome; these have been added by others. These anomalies include the absence of subcutaneous tissue; anterior rib abnormalities; subclavian venous aberrancies; and dysplasia of the latissimus dorsi, deltoid, external oblique, serratus, rectus abdominis, supraspinatus, and infraspinatus musculature. The most consistent of these components is partial or total absence of the pectoralis major muscle. Other additions to the constellation include upper extremity anomalies apart from syndactyly, including hypoplasia of the hand, forearm, and fingers (brachy-symphalangism). Breast anomalies, including amastia and hypomastia, have also been frequently associated with Poland syndrome. One of the main ways to approach breast reconstruction in these patients is to utilize the latissimus muscle. However, the latissimus dorsi muscle can be hypoplastic or even missing completely. This fact has rarely been reported in literature. In the few published reports in which the latissimus dorsi muscle was found to be hypoplastic, the precise extent of the latissimus deformity could not be judged adequately by physical examination alone because the results of the usual tests for functioning of the muscle (ie, the resistance test, the scapula test, the cough test) were normal. During these operations, however, the latissimus dorsi muscle was found to be grossly attenuated or highly hypoplastic. Poland syndrome occurs in a predominantly sporadic fashion; however, familial kindreds have been infrequently identified. Associations with other congenital defects have been linked with Möbius syndrome, neuroblastoma, Wilms tumor, myelomeningocele, leukemia, and non-Hodgkin lymphoma. The exact cause of Poland syndrome is unknown, but it is most likely a vascular anomaly. The leading theory is that the manifestations result from subclavian artery ?kinking? during the sixth week of gestation, with subsequent hypoplasia leading to distal musculoskeletal malformation. This has been termed a subclavian artery supply disruption sequence and is supported by Merlob et al's evaluation of subclavian artery diameter and flow. They found that these values were consistently decreased by 50% in affected individuals.

A 25-year-old woman who is at 16 weeks' gestation has an elevated serum alpha-fetoprotein level. Follow-up ultrasound shows spina bifida with myelomeningocele. Which of the following is the most appropriate next step in management? A) Gadolinium-enhanced MRI to further delineate abnormalitiesB) Prenatal repair in the late third trimesterC) Repair at 1 week of ageD) Repair at 3 months of ageE) Repair within 48 hours of delivery

The correct response is Option E. Postnatal repair of myelomeningocele is performed within the first 48 hours of life. Later repair is associated with worse outcomes. Myelomeningocele is a protrusion of the meninges and spinal cord via a defect in the caudal neural tube. The higher the level of the defect, the more severe the associated abnormalities, such as bowel and bladder dysfunction, lower limb sensory and motor abnormalities, and structural abnormalities (eg, club feet). Central nervous system problems include hindbrain herniation (Chiari II malformation), tethered cord and hydrocephalus. MRI can be performed to further delineate fetal central nervous system abnormalities. Gadolinium is not given in pregnancy as it is associated with fetal morbidity and mortality. The neurologic abnormalities caused by myelomeningocele are described by the "two-hit" hypothesis. The failure of the neural tube to close (first hit) results in exposure of the neural elements to amniotic fluid (second hit). Prenatal closure is thought to improve neurologic outcomes by decreasing the duration of exposure of neural elements to amniotic fluid. The management of myelomeningocele study (MOMS) is the only randomized-controlled trial that evaluated the outcomes of fetal surgery for myelomeningocele. The trial was stopped early because of improved outcomes in the prenatal correction group. The biggest difference was in the incidence of shunt-dependent hydrocephalus, which was 82% in the postnatal group and 40% in the prenatal group. The prenatal group also had better outcomes for mental development and motor function. However, there was a higher incidence of preterm labor and uterine dehiscence at delivery in the prenatal surgery group. Prenatal repair is performed between 19 and 27 weeks' gestation. Cesarean delivery is the preferred method of delivery for fetuses with myelomeningocele at many centers. This is performed when the fetus reaches term (i.e., 37 weeks' gestation). Cesarean delivery avoids trauma to the neural tube, maintains an aseptic environment, and allows elective scheduling of meningomyelocele surgical correction.

During a ventral hernia repair and abdominal wall reconstruction, a surgeon wishes to achieve full primary fascial closure over an intraperitoneal mesh. However, the fascia is under tension and requires a component separation to achieve full closure with minimal tension. Component separation technique involves which of the following? A) Incision of the anterior rectus sheathB) Internal oblique aponeurotomyC) Ligation of periumbilical perforatorsD) Plication of the linea semilunarisE) Preservation of intercostal nerves

The correct response is Option E. Preservation of segmental intercostal nerves is a critical aspect of component separation. Component separation is a powerful technique used during abdominal wall reconstruction to advance the abdominal fascia towards the midline and allow for primary closure with reduced tension. A component separation is performed by making an incision longitudinally in the external oblique aponeurosis, just lateral to the linea semilunaris. Only the fascia is cut, not the external oblique muscle. This allows for advancement of a myofascial complex consisting of the rectus abdominus muscle, internal oblique muscle and fascia, and transversalis muscle and fascia. The segmental intercostal nerves that supply the anterior abdominal wall run between the internal oblique and the transversalis muscle. They are deeper than the incision and dissection plane with component separation, and are thus protected from injury. This maintains abdominal wall tone in patients undergoing this procedure. External oblique aponeurotomy, not internal oblique aponeurotomy, is involved in component separation. Incision of the anterior rectus sheath is not part of component separation, although some modifications include incision of the posterior rectus sheath to release more fascia available for primary closure. Plication of the linea semilunaris is not a part of component separation. Ligation of periumbilical perforators is commonly performed in the standard, open approach to component separation, but it is not considered a critical component of the procedure, as it is possible to perform a perforator-sparing component separation.

A 55-year-old woman with recurrent rectal cancer comes to the office for preoperative consultation for pelvic exenteration, which will include total vaginal resection and reconstruction with a vertical rectus musculocutaneous flap. Which of the following long-term complications is most likely in this patient? A) Abdominal herniaB) Pelvic abscessC) Rectovaginal fistulaD) Small-bowel obstructionE) Vaginal stenosis

The correct response is Option E. Reconstruction of the vagina after oncologic resection can be challenging for plastic surgeons, as restoration of form and function must be considered. Restoration of form is often achievable by a variety of procedures, including the vertical rectus musculocutaneous flap, gracilis musculocutaneous flap, pudendal artery fasciocutaneous flap, skin grafting, and interpositional colonic grafts. Restoration of functional outcome, however, is controversial. Return of sexual activity across multiple studies shows wide variability from 31 to 100% (53.8% pooled data). A recent survey study found that 50% of responders were sexually active and were able to achieve orgasm after reconstruction. However, a majority of responders reported wishing that they had been given more information on the procedure before surgery. The most common long-term complication from total reconstruction of the vagina is stenosis with rates reported from 18 to 22%. This requires regular use of obturators for dilatation and this should be clearly discussed with patients preoperatively. Abdominal hernia, rectovaginal fistula, pelvic abscess, and small-bowel obstruction are less common complications.

An 18-year-old woman presents for surgical repair of an under-developed left breast. On physical examination, the left side shows concave chest wall, absent anterior axillary fold, and a hypoplastic and superiorly displaced nipple. The left upper extremity is most likely to demonstrate which of the following findings in this patient? A) AcrosyndactylyB) ArachnodactylyC) MacrodactylyD) PolydactylyE) Symbrachydactyly

The correct response is Option E. The patient described meets criteria for Poland syndrome. This complex includes breast and/or nipple-areola complex hypoplasia, absence of the sternal head of the pectoralis major, rib abnormalities, and syndactyly or symbrachydactyly, fused and shortened digits. Polydactyly (multiplication of digits), macrodactyly (overgrowth and enlarged digits), arachnodactyly (long fingers), and acrosyndactyly (fusion of distal aspects of digits only) are not usually associated with Poland syndrome.

Incomplete involution of the mammary ridge during embryonic development is most likely to result in which of the following? A) AmastiaB) GynecomastiaC) Inverted nippleD) Poland syndromeE) Polymastia

The correct response is Option E. The breast develops as the result of bilateral thickening of ectoderm along the milk line, or mammary ridge, from the axillary to the inguinal region. Mammary buds begin to develop as growths within the epidermis and invade the deeper mesenchyme. Much of the ridge disappears as the embryo develops as the result of apoptosis, except for the primary buds in the pectoral regions. Failure of regression of the mammary ridge can result in accessory breasts (polymastia) or accessory nipples (polythelia). Accessory breast tissue occurs in 1 to 2% of live births and commonly occurs in the axillae. Amastia is the complete absence of the mammary gland. This occurs due to either the failure of the mammary ridge to develop or the complete involution of the mammary ridge. Gynecomastia is defined as benign enlargement of the male breast. While pathologic cases can exist, it is most typically due to a normal response of the breast tissue to circulating levels of estrogen. Inverted nipples are due to failure of the mesenchyme to proliferate above the level of the skin. Poland syndrome can have the following components: hypoplasia of the breast and nipple, absence of the sternocostal portion of the pectoralis major muscle, absence of the pectoralis minor muscle, abnormalities of the chest wall, and anomalies of the upper extremity. Many etiologies have been hypothesized, with the most widely accepted being an interruption of the embryonic blood supply to the upper limb.

A 16-year-old girl comes to the clinic because she is dissatisfied with the asymmetric appearance of her breasts. Physical examination shows the right nipple-areola complex is more superiorly located and the breast volume is small. There is absence of the right anterior axillary fold. Which of the following other physical examination findings is most likely? A) Accessory nippleB) MicrotiaC) Right clubfootD) Scars consistent with repaired cleft lipE) Shortened right-hand digits

The correct response is Option E. The chest findings described are consistent with Poland syndrome with absence of the sternocostal head of the pectoralis major muscle. Poland syndrome can be associated with hand abnormalities, including shortened digits. Cleft lip, accessory nipple, clubfoot, and microtia are not known to be associated with Poland syndrome.

During fetal development, failure of the mammary pit to elevate above the skin level results in which of the following deformities? A) Accessory nippleB) AmastiaC) AmaziaD) AtheliaE) Nipple inversion

The correct response is Option E. The congenitally inverted nipple is common and occurs in 4% of infants. It results from the failure of the mesenchyme to proliferate above the level of the skin.Amastia is absence of the entire breast. Athelia is absence of the nipple. Amazia is absence of the mammary gland with an intact nipple and areola. Polythelia, or accessory nipple, results from failure of regression along the milk line.

A 29-year-old woman comes to the office because of a firm, mildly tender, well-circumscribed mass of the abdomen. A photograph is shown. The mass has been slowly increasing in size for the past 6 months. CT scan shows a mass that occupies the left musculofascial abdominal wall, including the rectus, external, and internal oblique muscles, and penetrates through the anterior rectus sheath. Which of the following is the most appropriate management? A) Neoadjuvant chemotherapyB) Radiation therapyC) Wide local excision, bilateral component separation, adjuvant chemotherapyD) Wide local excision, mesh placement, radiation therapyE) Wide local excision, right component separation, mesh reinforcement

The correct response is Option E. The lesion in this patient is a desmoid tumor, also known as aggressive fibromatosis. It is a benign tumor, usually found in younger patients between 10 and 40 years old, and is locally aggressive. It is oftentimes associated with pregnancy and prior surgery, and can frequently recur. Treatment is en bloc full-thickness wide local excision (usually with frozen section confirmation of negative margins). As aggressive full-thickness abdominal wall resection is standard of care, reconstruction is more challenging. If midline fascia can be reapproximated, it should be, as primary fascial closure is associated with the lowest hernia recurrence rates. Reinforcement with mesh has been prospectively demonstrated to reduce recurrence rates even further, especially in defects over 4 cm. If midline fascial reapproximation is not possible, reduction in the size of the defect is crucial to decrease recurrence rates. This is done by component separation. However, in this case, only a right component separation is possible, given that the tumor has invaded the left rectus muscle and obliques, precluding their use for myofascial advancement. If the obliques had been spared, a component separation could still have been attempted even if there were violation of the rectus. The most durable reconstruction would be achieved if midline fascial reapproximation were possible with mesh reinforcement. Second best would be reduction in the size of the defect with a right component separation and placement of mesh as a bridging underlay.There is no role for neoadjuvant chemotherapy or radiation therapy in the treatment of these tumors.

An 18-year-old woman comes to the office because of a small area of titanium extruding through the skin overlying the sacrum 12 months after undergoing a successful posterior spinal fusion and an autologous bone graft for myelodysplasia and lumbosacral kyphotic deformity. Physical examination shows a midline scar with a 2 × 2-cm wound at the level of the fourth lumbar vertebra with a visible screw. Which of the following is the most appropriate next step in management? A) Bilateral skin advancement flapsB) Free rectus abdominis muscle flapC) Gluteus maximus muscle flapD) Negative pressure wound therapy and skin graftE) Removal of the hardware

The correct response is Option E. The most appropriate next step in management of the exposed titanium is removal of the hardware. The titanium rod or one of the screws likely loosened and subsequently migrated to extrude through the skin. The vertebral column is adequately fused 1 year following the procedure. The orthopedics team should be consulted regarding removal of the hardware. Once the hardware is removed, the wound is easily repaired using delayed primary closure or by allowing it to heal secondarily. Although local skin flaps, skin grafts, regional muscle transfer, and free flaps will provide vascularized soft-tissue coverage over the exposed hardware, these options ultimately will fail. The exposed hardware is much stronger than any soft-tissue coverage and will extrude through a flap, especially when the patient is supine. Definitive management requires eliminating the cause of the problem, which is malpositioned hardware, and not inadequate soft-tissue coverage.

A 75-year-old woman with a history of right mastectomy and irradiation therapy presents with a sarcoma that requires radical resection and partial sternectomy. A photograph is shown. A pedicled flap is planned to repair the defect. Which of the following arteries supplies the most appropriate flap in this situation? A) Deep inferior epigastricB) Internal mammaryC) Lateral thoracicD) ThoracoacromialE) Thoracodorsal

The correct response is Option E. The most appropriate pedicled flap for this particular defect is a latissimus flap, shown in the photograph, supplied by the thoracodorsal artery. The latissimus flap is a Mathes/Nahai type V flap that can be transferred on its dominant pedicle (thoracodorsal artery) or on multiple segmental paraspinal perforators. The internal mammary artery terminates as the superior epigastric artery, which would provide blood supply for a superiorly based vertical rectus flap. The rectus flap is a Mathes/Nahai type III flap, with two dominant pedicles. However, this pedicle is not available because of the radical resection and prior irradiation. The lateral thoracic artery is one of two dominant pedicles supplying the serratus anterior muscle (Mathes/Nahai type III). This flap is an option, but it would not provide enough bulk necessary for the defect in this situation. The deep inferior epigastric artery (DIEA) supplies the DIEA perforator flap, which would be an option as a free tissue transfer but not as a pedicle flap for this situation.

A 43-year-old woman with a history of ovarian cancer is evaluated because of recurrence in the incision and bladder dome. History includes resection, chemotherapy, and radiation therapy. The gynecologic surgeon plans a wide resection of skin, fascia, bilateral lower rectus muscles, and bladder dome resulting in a 10 × 16-cm defect in the central lower abdomen. Which of the following is the most appropriate procedure to achieve abdominal closure? A) Bilateral component separationB) Primary closure onlyC) Primary closure with interpositional meshD) Use of a free latissimus muscle flapE) Use of a pedicled anterolateral thigh flap

The correct response is Option E. The patient described will have a 10 × 16-cm, full-thickness defect of the abdominal wall in the setting of previous radiation. A full-thickness defect this large in the setting of past radiation makes primary closure alone and primary closure with interpositional mesh a poor choice. Bilateral component separation will not be possible because the rectus muscles will be resected with the specimen. A pedicled anterolateral thigh flap will offer well-vascularized, nonradiated tissue that will include soft-tissue coverage as well as fascia for abdominal wall reconstruction. A free latissimus muscle flap, while providing well-vascularized tissue, would involve the increased risks of microsurgery and not provide fascia for closure. Therefore, Option E is most appropriate.

A 67-year-old woman with a large ventral hernia after prior exploratory laparotomy for trauma is scheduled for complex abdominal hernia repair with posterior components separation and mesh. The planned surgical technique involves division of which of the following muscles and locations? A) External abdominal oblique lateral to the linea semilunarisB) External abdominal oblique medial to the linea semilunarisC) Internal abdominal oblique lateral to the linea semilunarisD) Transversus abdominis lateral to the linea semilunarisE) Transversus abdominis medial to the linea semilunaris

The correct response is Option E. The posterior components separation technique involves dissection in the retro-rectus plane to release the transversus abdominis muscle medial to the linea semilunaris, allowing medialization of the posterior rectus sheath-transversalis fascia complex. This fascial layer is approximated to form the posterior layer of closure. The anterior layer, consisting of abdominal wall musculature, is approximated at the linea alba. Mesh is usually placed in between these two layers. In the more commonly performed anterior components separation technique, the external abdominal oblique is released lateral to the linea semilunaris to permit medialization of the rectus abdominis muscle for abdominal closure. The internal abdominal oblique is not released in anterior component separation. The internal abdominal oblique fascia is released medial to the semilunaris in posterior component separation.

A 46-year-old woman undergoes breast reconstruction with a pedicled transverse rectus abdominis musculocutaneous (TRAM) flap. Weight is 185 lb (84 kg); BMI is 32 kg/m2. The donor site is reconstructed with mesh. Six months postoperatively, the patient develops a bulge at the donor site. Which of the following best represents the deepest layer of the anterior rectus sheath when it is harvested caudal to the arcuate line? A) External oblique aponeurosisB) Internal oblique aponeurosisC) Rectus abdominis fasciaD) Transversalis fasciaE) Transversus abdominis aponeurosis

The correct response is Option E. The rectus sheath is the semifibrous compartment that encompasses the rectus abdominis muscle and consists of an anterior and posterior sheath created by the aponeurotic extensions of the external oblique, internal oblique, and transversus abdominis muscles. Proximal to the arcuate line, which is located approximately at the level of the anterior superior iliac spine, the anterior rectus sheath comprises the aponeuroses of the external oblique and the anterior leaf of the internal oblique. The internal oblique has two leaves, the deeper of which contributes to the posterior rectus sheath along with the transversus abdominis and transversalis fascia. At the level of the arcuate line, the posterior leaf of the internal oblique aponeurosis and the transversus abdominis aponeurosis travel superficially to the rectus abdominis, making the posterior sheath very weak. When the anterior rectus sheath is defective at this level, the likelihood for bulging or hernia is high. The level of the arcuate line is not always clear from topographical landmarks, which can make the planning of a transverse rectus abdominis musculocutaneous (TRAM) flap more difficult and increase the risk for a hernia complication.

A 24-year-old woman is scheduled to undergo correction of the defect shown in the photographs. Physical examination shows an elevated inframammary fold and herniation of breast tissue through the areolar complex. Which of the following is the most appropriate operative plan? A) Latissimus dorsi musculocutaneous flaps and placement of silicone gel prosthesesB) Subfascial placement of silicone gel prostheses with mastopexyC) Subglandular placement of silicone gel prosthesesD) Submuscular placement of silicone gel prosthesesE) Submuscular placement of silicone gel prostheses with mastopexy and scoring of the gland

The correct response is Option E. Tuberous breast deformity is a rare congenital condition that results in aberrant breast shape due to a constricting ring at the breast base. Abnormal development results in breast tissue deficiency, herniation of breast tissue into the nipple-areola complex, areolar enlargement, and breast asymmetry. Although latissimus flaps could be used for severe primary cases or reoperative secondary cases, such an aggressive intervention would not be warranted in the patient described. Surgical correction is challenging, but it can be achieved in a single-stage operation. This procedure should include submuscular placement of silicone or saline prostheses, mastopexy with areolar reduction, and scoring of the gland to relieve the constricting tissue. Postoperative results are depicted in the photograph shown.

Absence of which of the following structures is most characteristic of patients with Poland syndrome? A) Clavicular head of the pectoralis major muscleB) Ipsilateral ribsC) Pectoralis minor muscleD) Serratus anterior muscleE) Sternocostal head of the pectoralis major muscle

The correct response is Option E. What follows is the description from the original records found in Paris, France: "In 1841, Alfred Poland, a 19-year-old medical student at Guy's Hospital in London, England, published the classic description of the syndrome that bears his name. Serving as an anatomy demonstrator, Poland recorded that, in his subject: 'the whole of the sternal and costal portions of the pectoralis-major muscle were deficient; but its clavicular origin quite normal. In the left hand, the middle phalanges were absent. The web between the fingers extended to the first articulation the hand was shorter than the right the left thumb was quite normal.' In a footnote, he indicated that 'the hand has been deposited in the Museum of Guy's Hospital.' Poland was a popular teacher and later pursued a distinguished surgical career." (Charlier P, Deo S, Galassi FM, Benmoussa N. Poland syndrome before Alfred Poland: the oldest medical description [Paris, France, 1803]. Surg Radiol Anat. 2019;41[10]: 1117-1118.) Today, Poland syndrome presents a spectrum of chest wall anomalies ranging from simple to complex. These deficiencies are largely cosmetic, with the most common (simple) form presenting as a unilateral absence of the sternocostal head of the pectoralis major muscle. The deformity can also be complex, with ipsilateral absence of ribs, axillary webbing, and foreshortening of the hemithorax. Reconstructive options for the chest wall depend on anatomical severity, gender, associated anomalies, and, of course, the patient's preference. Clavicular head of pectoralis major muscle is generally present in these patients. Pectoralis minor, absence of serratus anterior, and rib absence may or may not be present in patients with Poland syndrome.

A 55-year-old man presents for a large abdominal midline hernia repair. A component separation is planned with a posterior approach and a retrorectus mesh placement. Which of the following layers can be divided to provide further release and preserve the innervation to the rectus muscle? A) Anterior rectus sheathB) External obliqueC) Internal obliqueD) Transversalis fasciaE) Transversus abdominis

he correct response is Option E. In the posterior component separation approach for ventral hernia repair, transversus abdominis release (TAR) can provide further mobility and preserve the innervation to the rectus muscle. The posterior approach reinforces hernia repair with a sublay mesh placed between the rectus muscle and posterior sheath. The Rives-Stoppa approach is associated with a 3 to 6% recurrence rate. To avoid disruption of the segmental nerves to the rectus, classical dissection was limited medial to the linea semilunaris. This, however, limited the space and reserved this technique for small- to medium-sized hernias. To extend this dissection laterally for use in larger defects, either the internal oblique or the transversus abdominis muscle can be divided. Division of the internal oblique divides the nerves to the rectus muscle. Division of the transversus abdominis can preserve these nerves. With this technique, the anterior rectus sheath is preserved as well as the external oblique and transversalis fascia.


Ensembles d'études connexes

Chemosynthesis and photosynthesis

View Set

Economics - Labor and Unemplyment

View Set

Chapter 2: Physical Quantities and Measurements

View Set